What's a plausible-sounding technobabble explanation for Mars to be a forest world supplying Earth with timber?

The name of the pictureThe name of the pictureThe name of the pictureClash Royale CLAN TAG#URR8PPP











up vote
12
down vote

favorite












In the story I want to tell, it's a thousand years in the future and Mars has been terraformed into a forest world (something like the forest-moon of Endor). It's settled in a few small towns and mostly populated by a rugged group of lumberjack types. Every year (Martian year) the logging crews go out in the fall, cut timber all winter, and in the spring the logs are shipped back to earth to be made into furniture, paper, etc. Earth is densely populated, urban, with all the modern conveniences, but has few large forests and is unwilling to harvest them.



My worldbuilding problem is that I want Mars to be a kind of hinterland with an uncivilized, frontier character. But if people have the technology to transport millions of pounds of lumber off Mars and land it safely on Earth, via antigravity drives or something, then Mars would not remain sparsely populated for long. Billions would move there and build giant cities complete with shopping malls and everything else. Moreover, if it were easy to transport heavy loads from Earth to Mars, the lumberjacks themselves would probably be replaced by heavy machinery such as real loggers use in the 21st century.



Instead, I want it to be very difficult and expensive for freight to get from Earth to Mars, so it'll mostly just be loggers with hand tools going that way each autumn, but very easy for heavy freight to be transported from Mars to Earth. The not-so-subtle metaphor is that Earth is "downstream" from Mars, and the logs are being "floated downriver" as in the olden days (on Earth) when there were annual riverdrives from logging operations in the deep woods downriver to the mills.



What's a plausible science-fiction explanation for this scenario? I'm defining "plausible" as meaning that the explanation must be internally consistent, i.e. not creating side effects such as godlike technological powers. Bonus points if your explanation also helps explain how Mars was terraformed.







share|improve this question






















  • I can easily picture flannel shirted lumberjacks singing the Monty Python Lumberjack song.....youtube.com/watch?v=sZa26_esLBE. Canada Vignettes: Log Driver's Waltz: youtube.com/watch?v=upsZZ2s3xv8. Enjoy
    – Thucydides
    Aug 10 at 5:47










  • Joe this question is inherently opinion based. A "good" explanation is completely subjective given that we have, for example, no idea what technology is available nor what constraints you would use to judge an answer as better.
    – James♦
    Aug 10 at 6:39










  • Answer-ers please keep on/off topic considerations in mind before answering.
    – James♦
    Aug 10 at 6:40










  • @James I have edited out the word "better" and replaced it with a specific definition of what I mean by "plausible", a more objective criterion for answers.
    – Joe
    Aug 10 at 21:59














up vote
12
down vote

favorite












In the story I want to tell, it's a thousand years in the future and Mars has been terraformed into a forest world (something like the forest-moon of Endor). It's settled in a few small towns and mostly populated by a rugged group of lumberjack types. Every year (Martian year) the logging crews go out in the fall, cut timber all winter, and in the spring the logs are shipped back to earth to be made into furniture, paper, etc. Earth is densely populated, urban, with all the modern conveniences, but has few large forests and is unwilling to harvest them.



My worldbuilding problem is that I want Mars to be a kind of hinterland with an uncivilized, frontier character. But if people have the technology to transport millions of pounds of lumber off Mars and land it safely on Earth, via antigravity drives or something, then Mars would not remain sparsely populated for long. Billions would move there and build giant cities complete with shopping malls and everything else. Moreover, if it were easy to transport heavy loads from Earth to Mars, the lumberjacks themselves would probably be replaced by heavy machinery such as real loggers use in the 21st century.



Instead, I want it to be very difficult and expensive for freight to get from Earth to Mars, so it'll mostly just be loggers with hand tools going that way each autumn, but very easy for heavy freight to be transported from Mars to Earth. The not-so-subtle metaphor is that Earth is "downstream" from Mars, and the logs are being "floated downriver" as in the olden days (on Earth) when there were annual riverdrives from logging operations in the deep woods downriver to the mills.



What's a plausible science-fiction explanation for this scenario? I'm defining "plausible" as meaning that the explanation must be internally consistent, i.e. not creating side effects such as godlike technological powers. Bonus points if your explanation also helps explain how Mars was terraformed.







share|improve this question






















  • I can easily picture flannel shirted lumberjacks singing the Monty Python Lumberjack song.....youtube.com/watch?v=sZa26_esLBE. Canada Vignettes: Log Driver's Waltz: youtube.com/watch?v=upsZZ2s3xv8. Enjoy
    – Thucydides
    Aug 10 at 5:47










  • Joe this question is inherently opinion based. A "good" explanation is completely subjective given that we have, for example, no idea what technology is available nor what constraints you would use to judge an answer as better.
    – James♦
    Aug 10 at 6:39










  • Answer-ers please keep on/off topic considerations in mind before answering.
    – James♦
    Aug 10 at 6:40










  • @James I have edited out the word "better" and replaced it with a specific definition of what I mean by "plausible", a more objective criterion for answers.
    – Joe
    Aug 10 at 21:59












up vote
12
down vote

favorite









up vote
12
down vote

favorite











In the story I want to tell, it's a thousand years in the future and Mars has been terraformed into a forest world (something like the forest-moon of Endor). It's settled in a few small towns and mostly populated by a rugged group of lumberjack types. Every year (Martian year) the logging crews go out in the fall, cut timber all winter, and in the spring the logs are shipped back to earth to be made into furniture, paper, etc. Earth is densely populated, urban, with all the modern conveniences, but has few large forests and is unwilling to harvest them.



My worldbuilding problem is that I want Mars to be a kind of hinterland with an uncivilized, frontier character. But if people have the technology to transport millions of pounds of lumber off Mars and land it safely on Earth, via antigravity drives or something, then Mars would not remain sparsely populated for long. Billions would move there and build giant cities complete with shopping malls and everything else. Moreover, if it were easy to transport heavy loads from Earth to Mars, the lumberjacks themselves would probably be replaced by heavy machinery such as real loggers use in the 21st century.



Instead, I want it to be very difficult and expensive for freight to get from Earth to Mars, so it'll mostly just be loggers with hand tools going that way each autumn, but very easy for heavy freight to be transported from Mars to Earth. The not-so-subtle metaphor is that Earth is "downstream" from Mars, and the logs are being "floated downriver" as in the olden days (on Earth) when there were annual riverdrives from logging operations in the deep woods downriver to the mills.



What's a plausible science-fiction explanation for this scenario? I'm defining "plausible" as meaning that the explanation must be internally consistent, i.e. not creating side effects such as godlike technological powers. Bonus points if your explanation also helps explain how Mars was terraformed.







share|improve this question














In the story I want to tell, it's a thousand years in the future and Mars has been terraformed into a forest world (something like the forest-moon of Endor). It's settled in a few small towns and mostly populated by a rugged group of lumberjack types. Every year (Martian year) the logging crews go out in the fall, cut timber all winter, and in the spring the logs are shipped back to earth to be made into furniture, paper, etc. Earth is densely populated, urban, with all the modern conveniences, but has few large forests and is unwilling to harvest them.



My worldbuilding problem is that I want Mars to be a kind of hinterland with an uncivilized, frontier character. But if people have the technology to transport millions of pounds of lumber off Mars and land it safely on Earth, via antigravity drives or something, then Mars would not remain sparsely populated for long. Billions would move there and build giant cities complete with shopping malls and everything else. Moreover, if it were easy to transport heavy loads from Earth to Mars, the lumberjacks themselves would probably be replaced by heavy machinery such as real loggers use in the 21st century.



Instead, I want it to be very difficult and expensive for freight to get from Earth to Mars, so it'll mostly just be loggers with hand tools going that way each autumn, but very easy for heavy freight to be transported from Mars to Earth. The not-so-subtle metaphor is that Earth is "downstream" from Mars, and the logs are being "floated downriver" as in the olden days (on Earth) when there were annual riverdrives from logging operations in the deep woods downriver to the mills.



What's a plausible science-fiction explanation for this scenario? I'm defining "plausible" as meaning that the explanation must be internally consistent, i.e. not creating side effects such as godlike technological powers. Bonus points if your explanation also helps explain how Mars was terraformed.









share|improve this question













share|improve this question




share|improve this question








edited Aug 10 at 21:58

























asked Aug 9 at 19:25









Joe

2,2451518




2,2451518











  • I can easily picture flannel shirted lumberjacks singing the Monty Python Lumberjack song.....youtube.com/watch?v=sZa26_esLBE. Canada Vignettes: Log Driver's Waltz: youtube.com/watch?v=upsZZ2s3xv8. Enjoy
    – Thucydides
    Aug 10 at 5:47










  • Joe this question is inherently opinion based. A "good" explanation is completely subjective given that we have, for example, no idea what technology is available nor what constraints you would use to judge an answer as better.
    – James♦
    Aug 10 at 6:39










  • Answer-ers please keep on/off topic considerations in mind before answering.
    – James♦
    Aug 10 at 6:40










  • @James I have edited out the word "better" and replaced it with a specific definition of what I mean by "plausible", a more objective criterion for answers.
    – Joe
    Aug 10 at 21:59
















  • I can easily picture flannel shirted lumberjacks singing the Monty Python Lumberjack song.....youtube.com/watch?v=sZa26_esLBE. Canada Vignettes: Log Driver's Waltz: youtube.com/watch?v=upsZZ2s3xv8. Enjoy
    – Thucydides
    Aug 10 at 5:47










  • Joe this question is inherently opinion based. A "good" explanation is completely subjective given that we have, for example, no idea what technology is available nor what constraints you would use to judge an answer as better.
    – James♦
    Aug 10 at 6:39










  • Answer-ers please keep on/off topic considerations in mind before answering.
    – James♦
    Aug 10 at 6:40










  • @James I have edited out the word "better" and replaced it with a specific definition of what I mean by "plausible", a more objective criterion for answers.
    – Joe
    Aug 10 at 21:59















I can easily picture flannel shirted lumberjacks singing the Monty Python Lumberjack song.....youtube.com/watch?v=sZa26_esLBE. Canada Vignettes: Log Driver's Waltz: youtube.com/watch?v=upsZZ2s3xv8. Enjoy
– Thucydides
Aug 10 at 5:47




I can easily picture flannel shirted lumberjacks singing the Monty Python Lumberjack song.....youtube.com/watch?v=sZa26_esLBE. Canada Vignettes: Log Driver's Waltz: youtube.com/watch?v=upsZZ2s3xv8. Enjoy
– Thucydides
Aug 10 at 5:47












Joe this question is inherently opinion based. A "good" explanation is completely subjective given that we have, for example, no idea what technology is available nor what constraints you would use to judge an answer as better.
– James♦
Aug 10 at 6:39




Joe this question is inherently opinion based. A "good" explanation is completely subjective given that we have, for example, no idea what technology is available nor what constraints you would use to judge an answer as better.
– James♦
Aug 10 at 6:39












Answer-ers please keep on/off topic considerations in mind before answering.
– James♦
Aug 10 at 6:40




Answer-ers please keep on/off topic considerations in mind before answering.
– James♦
Aug 10 at 6:40












@James I have edited out the word "better" and replaced it with a specific definition of what I mean by "plausible", a more objective criterion for answers.
– Joe
Aug 10 at 21:59




@James I have edited out the word "better" and replaced it with a specific definition of what I mean by "plausible", a more objective criterion for answers.
– Joe
Aug 10 at 21:59










13 Answers
13






active

oldest

votes

















up vote
12
down vote



accepted










Mars's atmosphere was largely carbon dioxide before; well, carbon dioxide's a great food source for plants, provided they have adequate sunlight! Plants' photosynthesis in turn could explain what methods were used to make Mars livable; all that carbon dioxide was trapped in the trees, and in turn, oxygen was pumped out into the atmosphere.



The more you log the trees and take their carbon off-planet (then plant some more), the higher the oxygen-to-carbon dioxide ratio gets (as the trees never die and rot and get consumed by carbon-dioxide-spewing fungi and bacteria). So, the forestation and logging of Mars could not just be the result of terraforming, but part of the process!






share|improve this answer




















  • Very clever, I like it!
    – Joe
    Aug 10 at 22:01










  • I've got good ideas from all of these answers, but can only accept one, so this is it. I like that you tied it in to the terraforming method.
    – Joe
    Aug 13 at 0:07










  • @Joe Thank you very much! I kind of had a eureka moment as I was typing it XD
    – Matthew Dave
    Aug 13 at 0:20

















up vote
20
down vote













Mars has a space elevator. Earth doesn't.



Space elevators are fantastic pieces of technology that make the trip from surface to orbit incredibly cheap and easy. They are unfortunately also incredibly dangerous as Earth learned when a terrorist group managed to sever the Curacao elevator in 2591. The resulting cataclysm killed hundreds of millions outright as the ~50,000 km long structure fell to Earth wrapping around the globe nearly 2 full times. A global ban on these inherently vulnerable structures was put into place shortly thereafter making any travel out of Earth’s gravity well using conventional chemical rockets extremely expensive. The fledgling Martian colony was cut off from support from Earth but soon found that as the sole power in the solar system in possession of a space elevator it wielded immense economic power. Immigration and trade from Earth to Mars slowed to a trickle, however, exports from Mars to Earth remained economically viable.






share|improve this answer


















  • 3




    While space elevators are horribly vulnerable it's only an economic catastrophe if one comes down. The cables aren't going to be heavy enough to do anything serious if they do come down (and that's assuming they don't simply burn as they fall, anyway.)
    – Loren Pechtel
    Aug 9 at 21:18






  • 8




    Any realistic elevator cable is going to be more ribbon-like than rope-like, so a severed cable isn't going to come crashing down. The reason Mars has a space elevator and Earth doesn't is much simpler: lower gravity. A space elevator on Earth requires technobabble materials to build, while a Martian elevator could probably be done using Kevlar or carbon fiber.
    – Mark
    Aug 9 at 21:28






  • 3




    I would caution that since we don’t know what material these elevators will be made of we shouldn’t discount the possibility that they will be heavy enough to do damage. Perhaps unlikely, but not implausible. Also worth noting that the cable will get thicker as you go up with the thickest portion lying in geosynchronous orbit where the tension is the greatest. @Mark’s suggestion that martian elevators are achievable while Earth elevators are out of reach is also an excellent explanation for the proposed situation.
    – Mike Nichols
    Aug 9 at 21:39






  • 2




    No terrorists needed -- 1000 years from now, materials science has advanced to the point building a space elevator on Mars is trivial, but on Earth is still practically impossible: physics.stackexchange.com/questions/33547/…
    – Alexander Duchene
    Aug 10 at 3:05










  • I understand some ideas about space elevators include an "anchor" at the top, like an asteroid in geosynchronous orbit. So perhaps that could have crashed.
    – Joe
    Aug 10 at 22:01

















up vote
7
down vote













Honestly, I don't see that you have a problem



How many people own yachts? And yet the cheapest form of transport on Earth is an ocean-going super freighter. So, from the perspective of people just jumping in their cars and moving, there's already the problem of the cost of the car spaceship, cost of fuel, and travel time. Your average Joe Schmoe could never afford the trip.



How to keep the wealthy off? That's easy, too. Mars property (somewhat like Hawaii) is 100% leased to corporate ventures under strict laws and oversight by a panel of United Earth nations who manage the resource. There isn't a square inch available for non-purpose habitation and the consequences are, well, let's call them breathtaking in their scope of civil punishment.



Note that you'll never have 100% compliance. You'll have the corporations creating resorts for their people, friends, and "associates." All well within the UE rules, of course. But that kind of nearly meaninless population isn't, I think, what you're talking about.



Finally, just in case you decide to make space travel cheap. If it is cheap, then so is the policing force needed to beat away the unwashed barbarians who don't want to live within spitting distance of their Terran neighbors.






share|improve this answer



























    up vote
    7
    down vote













    No technobabble required. Earth has a deep gravity well and a huge, thick atmosphere. Mars has a shallow gravity well, and presumably even after terraforming still has a thin atmosphere.



    Going from Mars to Earth is cheap (if a bit slow). A surface-mounted mass driver can throw packages of logs directly into an Earth-intercept trajectory. The thin atmosphere means you don't need much shielding on the way out (particularly if you're launching from the top of Olympus Mons), and Earth's thick atmosphere provides easy aerocapture and landing six months later.



    Going from Earth to Mars is much harder. The accelerations involved in a mass-driver launch make it unsuitable for anything but durable freight; humans and fragile goods need to be launched on chemical rockets. Either way, you're spending far more energy fighting gravity and drag than you would leaving Mars. At the other end of the six-month trip (which itself would tend to discourage human travel), the thin Martian atmosphere makes aerocapture and parachute/glider landing tricky. You'll need to haul engines and fuel with you, which further drives up the cost of travel.






    share|improve this answer



























      up vote
      6
      down vote













      Your antigravity drives or whatever you are using work only when the body has a bulk density comparable to wood, about $0.8 g/cm^3$. Anything denser than that will have to follow the chemical rocket path, with its related costs, efficiencies and limitations.



      Casually enough this will allow you to cheaply ship wood, but nothing more than that.






      share|improve this answer


















      • 2




        Potassium conveniently has a density comparable to wood(0.86), so if Mars does have ancient evaporated seas then an earlier potassium/potash/fertilizer boom could be the reason why Mars is forested in the first place.
        – Giter
        Aug 9 at 20:04







      • 4




        Fridge logic: Air is much less dense than wood, you should have no trouble shipping a load of steel if you only partially fill the spacecraft...
        – Brilliand
        Aug 9 at 22:04






      • 1




        @Brilliand: Perfect, now I have an excuse to design heavier-than-air zeppelins.
        – Kevin
        Aug 10 at 0:58










      • @Brilliand, writing my answer I had in mind the bulk density, not the average density. I have now edited my answer to make it clear.
        – L.Dutch♦
        Aug 10 at 6:19

















      up vote
      4
      down vote













      Material limits. While we can't currently build a space elevator on Mars we can do the next best thing--elevators on Deimos and Phobos. (See here) Now, we don't need much more strength to get the low end of the Phobos elevator into the range that aircraft can intercept it.



      Presto, travel from Mars to Earth becomes cheap, the hardest part of the whole mission is protecting them from their trip through the fire coming down. However, the cables aren't strong enough to build an Earth-based elevator. Thus to go from Earth to Mars still has to be done the hard way on a tail of fire.



      You're not going to get it down to loggers with hand tools as the launch from Mars still needs high performance aircraft--but that's still an awful lot cheaper than rockets.






      share|improve this answer





























        up vote
        3
        down vote













        Terraforming without lots of tech.



        Later this year, astronomers discover a very large icy comet inbound from the Kuiper belt, destined to make impact with an Asteroid so that it scrubs most of the velocity relative to Mars. The remnants from this collision impact Mars 2 months later delivering many megatons of water, and other interesting volatiles such as carbon dioxide, carbon monoxide, and ammonia. Won't be pleasant on Mars for quite a while, but once things settle, you have the basic materials you need in place.



        Taking advantage of this event, a seed ship is sent to Mars after the fact to seed the new atmosphere with various designed to transform the atmosphere into something suitable for forestry.



        Still, Mars has trouble retaining an atmosphere, so NASA also prepares a big magnetic shield for the atmosphere to help prevent atmosphere from being stripped away by the solar wind.



        Making it comparatively cheap to send wood back to Earth requires non-fanciful technologies. A space elevator would be much easier to construct for Mars than Earth because of its lower gravity. Still won't be cheap to build, but you don't have to start construction for a long-time still -- waiting for better materials, construction techniques, etc.



        Significant improvements in orbital transfer (between Mars and Earth) are also needed. Careful use of light sails should make this reasonably priced in either direction as long as you are not in a hurry.



        Dropping wood from orbit to the Earth is not that difficult if you are willing to use the wood itself as ablative shielding.






        share|improve this answer




















        • I think the suggestion of building space elevators on mars is a good one - that's something much easier there than earth.
          – Tim B♦
          Aug 9 at 20:54

















        up vote
        3
        down vote













        The wood itself is the fuel for the rockets.



        In combination with jedmeyer's answer suggesting that the trees grow much larger than they would on Earth, perhaps there is a real chemical process or, failing that, something catalyzed by handwavium (mined only on Mars) that allows the wood itself to function as rocket fuel. Hollow out a large tree, fill it with oxidizer, and launch it. It could fire retrorockets on its approach to Earth and splash down in the ocean where it would be recovered by a fleet of logging ships.



        (I know this answer is absurd, but it amuses me greatly.)






        share|improve this answer




















        • Welcome to WorldBuilding.SE! We're glad you could join us! I don't know how to put links in my comments, or else I would invite you to click somewhere to learn about our rules and culture, or take a tour. I'm sure someone will fill that in for me. This is a fun answer :) The uses for handwavium are as abundant as the resource itself. Hope you enjoy your time here!
          – boxcartenant
          Aug 9 at 20:51










        • It seems like this would decrease both the quality and the quantity of the wood received, not to mention the wood would probably just burn up during reentry. But it is quite the creative idea.
          – Billy S
          Aug 9 at 23:06










        • @BillyS, wood is a decent material to make an ablative heat shield out of. It's considerably heavier than synthetic resin heatshields, which is why it never saw much use, but if you're already throwing logs from planet to planet...
          – Mark
          Aug 10 at 0:47










        • @Mark Really? I wouldn't have expected that, due to wood's flammable nature ... I guess I should do more research next time.
          – Billy S
          Aug 10 at 1:13







        • 1




          @BillyS, wood tends to form an outer layer of char that provides decent insulation and resists burning. It takes quite a bit of effort to actually burn a full-sized log.
          – Mark
          Aug 10 at 1:52

















        up vote
        2
        down vote













        The power of Economics!



        The solution you have here is that Mars has less gravity than Earth (3.7m/s^2 vs. (9.8 m/s^2). Because of this massive difference, the trees that grew up on Earth are able to grow MUCH taller on Mars, solving the scenario in two ways.



        1. The trees grow faster and taller due to less gravity. Thus, a frontier-like world could develop shortly after being seeded and "terraformed".

        2. Big Trees = More Wood. If you grow trees easier on Mars, Martian wood becomes more affordable: supply and demand. The shipping is just an overhead cost.

        Where the scenario gets more challenging is the "downstream" metaphor. It would be possible if you consider aerobraking, since Mars has a less dense atmosphere than Earth. Launching from Mars requires a little bit less to land on Earth thanks to its thick atmosphere than the reverse path. However, that makes it unlikely that megaflora could flourish due to the insufficient pressure.



        I recommend that you consider another cause for the unindustrialized world:



        -There isn't an established mining base on Mars for enough metal for automated machinery, and no one wants to ship that out when you get enough wood from the locals!



        -Overpopulation on Earth means more affordable labor for companies, and there's no room to plant large forests!






        share|improve this answer




















        • Part of what I'm thinking about to explain why few people go to Mars is that people on Earth will generally be wimps. The risk of splinters, the loud noises, the weather that isn't programmed on a schedule, will all seem too rugged to most of them.
          – Joe
          Aug 10 at 22:13

















        up vote
        2
        down vote













        I like @Mark's mass driver for logs - bare logs flying thru space.



        My idea for the cheap low tech terraforming - sometime in the mid 21st century the popular prophet inheriting the American government (and its nuclear stock) is told by God that nuking the Martian polar icecaps will bring great rewards to humanity. The Americans proceed to do exactly that, launching hundreds of ICBMs to detonate on Mars. The nukes are followed with arklike rockets of seeds and spores.



        The scheme works like a charm, liberating immense stores of water and CO2 which cause a runaway greenhouse effect and melt the rest of the immense subsurface stores of these gases. Certain seeds among those sent up give rise to the forests which are thriving 1000 years later.






        share|improve this answer



























          up vote
          2
          down vote













          Mars has lower gravity than Earth, so it's already easier to achieve escape velocity.



          However, with all of Mars as a giant forest, the atmosphere is now much more oxygen-rich than Earth. This means that producing liquid oxygen fuel is quicker and cheaper, and any craft with an air-intake can burn fuel much more efficiently than one on Earth.



          In that circumstance, Earth really would be downstream. The freighter's easily lift into Mars orbit, burn the rest of their fuel to escape orbit at just the right spot...and free-fall into a precise Earth orbit where "local" spacecraft can retrieve them and bring them down.



          You could even add a touch of danger to the retrieval: if the freight isn't caught and slowed on the first pass, it'll be going too fast, and will slingshot into empty space. If a corp misses a shipment at the prime capture point, nearby opportunists could turn a hefty profit by claiming the freight later in its trajectory--when it's much more dangerous to make the attempt.






          share|improve this answer



























            up vote
            2
            down vote













            Monsanto got chased off of Earth



            Monsanto is one of the most hated big corporations on this planet presently. Not because their products are actually bad; genetically modified crops are safe and pretty much ubiquitous by now. But Monsanto's business practices has every ideologically green party and ditto non governmental organisation getting their knickers in a big itchy twist. And somewhat rightfully so because Monsanto is really pushing the limits when it comes to copyrights, trying to control what people can do with their products long after the product has been bought and used by the client.



            So Monsanto — or its 3018 equivalent — have set up shop on Mars instead. And they are doing quite well for themselves! Their radiation hardened planktons and tree products have been instrumental in terraforming Mars without having to bother with restoring the planet's magnetic field. And not only that but the wood is amazing, being (insert qualities that are great for wood to have here). There is no-one that has those species of trees growing anywhere, thanks to Monsanto's quite excellent gene-hackers/engineers.



            Of course there is the slight issue on how to protect such a valuable product and not have it spread like weed all over the solar system and neighbouring stars. So the genetic code is written such that it requires Mars conditions — the right gravity, the right radiation levels, the right atmosphere — or the seeds / seedlings simply will not catch. And Monsanto are annoyingly good at this sort of genetic copyright protection.



            Too bad for them then, when an ideologically green terrorist organisation infiltrated the Ice Miner's Guild and in a brilliant move bot-net'd an entire flotilla of harvest drones to annihilate every Monsanto office on Mars by dropping asteroid ice bergs on them. (Side plot hook: what happens when you take a genetic engineering corporation, physically smash it to bits and let out all their lab samples and secret projects on a young booming forest planet)



            So there you have it: a — now defunct — genetics engineering corporation created a marvellous but very Mars specific species of wood... and then they got wiped out. Now their products keep growing on Mars, and are ripe for harvest.






            share|improve this answer





























              up vote
              1
              down vote













              Terraforming Mars: solar-powered artificial magnetosphere, water from Ceres (Mars is downstream from Ceres and ceres lower gravity makes it easy to fly water to Mars), nitrogen from Titan's atmosphere to start the Nitrogen cycle. In the long term, some fertilizers in mineral form (and additional Carbon Dioxide) must be replenished in one way or another.



              Environmental issues: too much damage to ecosystems on Earth is a good motive to turn Mars into a huge nature reserve with plants, wildlife and other organisms. Migration to Mars is restricted to park rangers, loggers and other related jobs, so that population is minimal. Some area is designated for logging.



              Self-government: the new Martians take control of their business. Machinery is restricted to less sophisticated tools, but cargo spaceships are still indispensable. Park rangers may double as loggers in the spare time and further reduce the need for more workers. The "martians" may actually make lots of profits from managing this business, and controlling who & how others join is a way to keep much of the profit to themselves. They may windowdress their management decisions with environmentalist ideology.






              share|improve this answer






















                Your Answer




                StackExchange.ifUsing("editor", function ()
                return StackExchange.using("mathjaxEditing", function ()
                StackExchange.MarkdownEditor.creationCallbacks.add(function (editor, postfix)
                StackExchange.mathjaxEditing.prepareWmdForMathJax(editor, postfix, [["$", "$"], ["\\(","\\)"]]);
                );
                );
                , "mathjax-editing");

                StackExchange.ready(function()
                var channelOptions =
                tags: "".split(" "),
                id: "579"
                ;
                initTagRenderer("".split(" "), "".split(" "), channelOptions);

                StackExchange.using("externalEditor", function()
                // Have to fire editor after snippets, if snippets enabled
                if (StackExchange.settings.snippets.snippetsEnabled)
                StackExchange.using("snippets", function()
                createEditor();
                );

                else
                createEditor();

                );

                function createEditor()
                StackExchange.prepareEditor(
                heartbeatType: 'answer',
                convertImagesToLinks: false,
                noModals: false,
                showLowRepImageUploadWarning: true,
                reputationToPostImages: null,
                bindNavPrevention: true,
                postfix: "",
                noCode: true, onDemand: true,
                discardSelector: ".discard-answer"
                ,immediatelyShowMarkdownHelp:true
                );



                );








                 

                draft saved


                draft discarded


















                StackExchange.ready(
                function ()
                StackExchange.openid.initPostLogin('.new-post-login', 'https%3a%2f%2fworldbuilding.stackexchange.com%2fquestions%2f120907%2fwhats-a-plausible-sounding-technobabble-explanation-for-mars-to-be-a-forest-wor%23new-answer', 'question_page');

                );

                Post as a guest






























                13 Answers
                13






                active

                oldest

                votes








                13 Answers
                13






                active

                oldest

                votes









                active

                oldest

                votes






                active

                oldest

                votes








                up vote
                12
                down vote



                accepted










                Mars's atmosphere was largely carbon dioxide before; well, carbon dioxide's a great food source for plants, provided they have adequate sunlight! Plants' photosynthesis in turn could explain what methods were used to make Mars livable; all that carbon dioxide was trapped in the trees, and in turn, oxygen was pumped out into the atmosphere.



                The more you log the trees and take their carbon off-planet (then plant some more), the higher the oxygen-to-carbon dioxide ratio gets (as the trees never die and rot and get consumed by carbon-dioxide-spewing fungi and bacteria). So, the forestation and logging of Mars could not just be the result of terraforming, but part of the process!






                share|improve this answer




















                • Very clever, I like it!
                  – Joe
                  Aug 10 at 22:01










                • I've got good ideas from all of these answers, but can only accept one, so this is it. I like that you tied it in to the terraforming method.
                  – Joe
                  Aug 13 at 0:07










                • @Joe Thank you very much! I kind of had a eureka moment as I was typing it XD
                  – Matthew Dave
                  Aug 13 at 0:20














                up vote
                12
                down vote



                accepted










                Mars's atmosphere was largely carbon dioxide before; well, carbon dioxide's a great food source for plants, provided they have adequate sunlight! Plants' photosynthesis in turn could explain what methods were used to make Mars livable; all that carbon dioxide was trapped in the trees, and in turn, oxygen was pumped out into the atmosphere.



                The more you log the trees and take their carbon off-planet (then plant some more), the higher the oxygen-to-carbon dioxide ratio gets (as the trees never die and rot and get consumed by carbon-dioxide-spewing fungi and bacteria). So, the forestation and logging of Mars could not just be the result of terraforming, but part of the process!






                share|improve this answer




















                • Very clever, I like it!
                  – Joe
                  Aug 10 at 22:01










                • I've got good ideas from all of these answers, but can only accept one, so this is it. I like that you tied it in to the terraforming method.
                  – Joe
                  Aug 13 at 0:07










                • @Joe Thank you very much! I kind of had a eureka moment as I was typing it XD
                  – Matthew Dave
                  Aug 13 at 0:20












                up vote
                12
                down vote



                accepted







                up vote
                12
                down vote



                accepted






                Mars's atmosphere was largely carbon dioxide before; well, carbon dioxide's a great food source for plants, provided they have adequate sunlight! Plants' photosynthesis in turn could explain what methods were used to make Mars livable; all that carbon dioxide was trapped in the trees, and in turn, oxygen was pumped out into the atmosphere.



                The more you log the trees and take their carbon off-planet (then plant some more), the higher the oxygen-to-carbon dioxide ratio gets (as the trees never die and rot and get consumed by carbon-dioxide-spewing fungi and bacteria). So, the forestation and logging of Mars could not just be the result of terraforming, but part of the process!






                share|improve this answer












                Mars's atmosphere was largely carbon dioxide before; well, carbon dioxide's a great food source for plants, provided they have adequate sunlight! Plants' photosynthesis in turn could explain what methods were used to make Mars livable; all that carbon dioxide was trapped in the trees, and in turn, oxygen was pumped out into the atmosphere.



                The more you log the trees and take their carbon off-planet (then plant some more), the higher the oxygen-to-carbon dioxide ratio gets (as the trees never die and rot and get consumed by carbon-dioxide-spewing fungi and bacteria). So, the forestation and logging of Mars could not just be the result of terraforming, but part of the process!







                share|improve this answer












                share|improve this answer



                share|improve this answer










                answered Aug 9 at 20:02









                Matthew Dave

                3555




                3555











                • Very clever, I like it!
                  – Joe
                  Aug 10 at 22:01










                • I've got good ideas from all of these answers, but can only accept one, so this is it. I like that you tied it in to the terraforming method.
                  – Joe
                  Aug 13 at 0:07










                • @Joe Thank you very much! I kind of had a eureka moment as I was typing it XD
                  – Matthew Dave
                  Aug 13 at 0:20
















                • Very clever, I like it!
                  – Joe
                  Aug 10 at 22:01










                • I've got good ideas from all of these answers, but can only accept one, so this is it. I like that you tied it in to the terraforming method.
                  – Joe
                  Aug 13 at 0:07










                • @Joe Thank you very much! I kind of had a eureka moment as I was typing it XD
                  – Matthew Dave
                  Aug 13 at 0:20















                Very clever, I like it!
                – Joe
                Aug 10 at 22:01




                Very clever, I like it!
                – Joe
                Aug 10 at 22:01












                I've got good ideas from all of these answers, but can only accept one, so this is it. I like that you tied it in to the terraforming method.
                – Joe
                Aug 13 at 0:07




                I've got good ideas from all of these answers, but can only accept one, so this is it. I like that you tied it in to the terraforming method.
                – Joe
                Aug 13 at 0:07












                @Joe Thank you very much! I kind of had a eureka moment as I was typing it XD
                – Matthew Dave
                Aug 13 at 0:20




                @Joe Thank you very much! I kind of had a eureka moment as I was typing it XD
                – Matthew Dave
                Aug 13 at 0:20










                up vote
                20
                down vote













                Mars has a space elevator. Earth doesn't.



                Space elevators are fantastic pieces of technology that make the trip from surface to orbit incredibly cheap and easy. They are unfortunately also incredibly dangerous as Earth learned when a terrorist group managed to sever the Curacao elevator in 2591. The resulting cataclysm killed hundreds of millions outright as the ~50,000 km long structure fell to Earth wrapping around the globe nearly 2 full times. A global ban on these inherently vulnerable structures was put into place shortly thereafter making any travel out of Earth’s gravity well using conventional chemical rockets extremely expensive. The fledgling Martian colony was cut off from support from Earth but soon found that as the sole power in the solar system in possession of a space elevator it wielded immense economic power. Immigration and trade from Earth to Mars slowed to a trickle, however, exports from Mars to Earth remained economically viable.






                share|improve this answer


















                • 3




                  While space elevators are horribly vulnerable it's only an economic catastrophe if one comes down. The cables aren't going to be heavy enough to do anything serious if they do come down (and that's assuming they don't simply burn as they fall, anyway.)
                  – Loren Pechtel
                  Aug 9 at 21:18






                • 8




                  Any realistic elevator cable is going to be more ribbon-like than rope-like, so a severed cable isn't going to come crashing down. The reason Mars has a space elevator and Earth doesn't is much simpler: lower gravity. A space elevator on Earth requires technobabble materials to build, while a Martian elevator could probably be done using Kevlar or carbon fiber.
                  – Mark
                  Aug 9 at 21:28






                • 3




                  I would caution that since we don’t know what material these elevators will be made of we shouldn’t discount the possibility that they will be heavy enough to do damage. Perhaps unlikely, but not implausible. Also worth noting that the cable will get thicker as you go up with the thickest portion lying in geosynchronous orbit where the tension is the greatest. @Mark’s suggestion that martian elevators are achievable while Earth elevators are out of reach is also an excellent explanation for the proposed situation.
                  – Mike Nichols
                  Aug 9 at 21:39






                • 2




                  No terrorists needed -- 1000 years from now, materials science has advanced to the point building a space elevator on Mars is trivial, but on Earth is still practically impossible: physics.stackexchange.com/questions/33547/…
                  – Alexander Duchene
                  Aug 10 at 3:05










                • I understand some ideas about space elevators include an "anchor" at the top, like an asteroid in geosynchronous orbit. So perhaps that could have crashed.
                  – Joe
                  Aug 10 at 22:01














                up vote
                20
                down vote













                Mars has a space elevator. Earth doesn't.



                Space elevators are fantastic pieces of technology that make the trip from surface to orbit incredibly cheap and easy. They are unfortunately also incredibly dangerous as Earth learned when a terrorist group managed to sever the Curacao elevator in 2591. The resulting cataclysm killed hundreds of millions outright as the ~50,000 km long structure fell to Earth wrapping around the globe nearly 2 full times. A global ban on these inherently vulnerable structures was put into place shortly thereafter making any travel out of Earth’s gravity well using conventional chemical rockets extremely expensive. The fledgling Martian colony was cut off from support from Earth but soon found that as the sole power in the solar system in possession of a space elevator it wielded immense economic power. Immigration and trade from Earth to Mars slowed to a trickle, however, exports from Mars to Earth remained economically viable.






                share|improve this answer


















                • 3




                  While space elevators are horribly vulnerable it's only an economic catastrophe if one comes down. The cables aren't going to be heavy enough to do anything serious if they do come down (and that's assuming they don't simply burn as they fall, anyway.)
                  – Loren Pechtel
                  Aug 9 at 21:18






                • 8




                  Any realistic elevator cable is going to be more ribbon-like than rope-like, so a severed cable isn't going to come crashing down. The reason Mars has a space elevator and Earth doesn't is much simpler: lower gravity. A space elevator on Earth requires technobabble materials to build, while a Martian elevator could probably be done using Kevlar or carbon fiber.
                  – Mark
                  Aug 9 at 21:28






                • 3




                  I would caution that since we don’t know what material these elevators will be made of we shouldn’t discount the possibility that they will be heavy enough to do damage. Perhaps unlikely, but not implausible. Also worth noting that the cable will get thicker as you go up with the thickest portion lying in geosynchronous orbit where the tension is the greatest. @Mark’s suggestion that martian elevators are achievable while Earth elevators are out of reach is also an excellent explanation for the proposed situation.
                  – Mike Nichols
                  Aug 9 at 21:39






                • 2




                  No terrorists needed -- 1000 years from now, materials science has advanced to the point building a space elevator on Mars is trivial, but on Earth is still practically impossible: physics.stackexchange.com/questions/33547/…
                  – Alexander Duchene
                  Aug 10 at 3:05










                • I understand some ideas about space elevators include an "anchor" at the top, like an asteroid in geosynchronous orbit. So perhaps that could have crashed.
                  – Joe
                  Aug 10 at 22:01












                up vote
                20
                down vote










                up vote
                20
                down vote









                Mars has a space elevator. Earth doesn't.



                Space elevators are fantastic pieces of technology that make the trip from surface to orbit incredibly cheap and easy. They are unfortunately also incredibly dangerous as Earth learned when a terrorist group managed to sever the Curacao elevator in 2591. The resulting cataclysm killed hundreds of millions outright as the ~50,000 km long structure fell to Earth wrapping around the globe nearly 2 full times. A global ban on these inherently vulnerable structures was put into place shortly thereafter making any travel out of Earth’s gravity well using conventional chemical rockets extremely expensive. The fledgling Martian colony was cut off from support from Earth but soon found that as the sole power in the solar system in possession of a space elevator it wielded immense economic power. Immigration and trade from Earth to Mars slowed to a trickle, however, exports from Mars to Earth remained economically viable.






                share|improve this answer














                Mars has a space elevator. Earth doesn't.



                Space elevators are fantastic pieces of technology that make the trip from surface to orbit incredibly cheap and easy. They are unfortunately also incredibly dangerous as Earth learned when a terrorist group managed to sever the Curacao elevator in 2591. The resulting cataclysm killed hundreds of millions outright as the ~50,000 km long structure fell to Earth wrapping around the globe nearly 2 full times. A global ban on these inherently vulnerable structures was put into place shortly thereafter making any travel out of Earth’s gravity well using conventional chemical rockets extremely expensive. The fledgling Martian colony was cut off from support from Earth but soon found that as the sole power in the solar system in possession of a space elevator it wielded immense economic power. Immigration and trade from Earth to Mars slowed to a trickle, however, exports from Mars to Earth remained economically viable.







                share|improve this answer














                share|improve this answer



                share|improve this answer








                edited Aug 9 at 21:11

























                answered Aug 9 at 20:52









                Mike Nichols

                7,02952662




                7,02952662







                • 3




                  While space elevators are horribly vulnerable it's only an economic catastrophe if one comes down. The cables aren't going to be heavy enough to do anything serious if they do come down (and that's assuming they don't simply burn as they fall, anyway.)
                  – Loren Pechtel
                  Aug 9 at 21:18






                • 8




                  Any realistic elevator cable is going to be more ribbon-like than rope-like, so a severed cable isn't going to come crashing down. The reason Mars has a space elevator and Earth doesn't is much simpler: lower gravity. A space elevator on Earth requires technobabble materials to build, while a Martian elevator could probably be done using Kevlar or carbon fiber.
                  – Mark
                  Aug 9 at 21:28






                • 3




                  I would caution that since we don’t know what material these elevators will be made of we shouldn’t discount the possibility that they will be heavy enough to do damage. Perhaps unlikely, but not implausible. Also worth noting that the cable will get thicker as you go up with the thickest portion lying in geosynchronous orbit where the tension is the greatest. @Mark’s suggestion that martian elevators are achievable while Earth elevators are out of reach is also an excellent explanation for the proposed situation.
                  – Mike Nichols
                  Aug 9 at 21:39






                • 2




                  No terrorists needed -- 1000 years from now, materials science has advanced to the point building a space elevator on Mars is trivial, but on Earth is still practically impossible: physics.stackexchange.com/questions/33547/…
                  – Alexander Duchene
                  Aug 10 at 3:05










                • I understand some ideas about space elevators include an "anchor" at the top, like an asteroid in geosynchronous orbit. So perhaps that could have crashed.
                  – Joe
                  Aug 10 at 22:01












                • 3




                  While space elevators are horribly vulnerable it's only an economic catastrophe if one comes down. The cables aren't going to be heavy enough to do anything serious if they do come down (and that's assuming they don't simply burn as they fall, anyway.)
                  – Loren Pechtel
                  Aug 9 at 21:18






                • 8




                  Any realistic elevator cable is going to be more ribbon-like than rope-like, so a severed cable isn't going to come crashing down. The reason Mars has a space elevator and Earth doesn't is much simpler: lower gravity. A space elevator on Earth requires technobabble materials to build, while a Martian elevator could probably be done using Kevlar or carbon fiber.
                  – Mark
                  Aug 9 at 21:28






                • 3




                  I would caution that since we don’t know what material these elevators will be made of we shouldn’t discount the possibility that they will be heavy enough to do damage. Perhaps unlikely, but not implausible. Also worth noting that the cable will get thicker as you go up with the thickest portion lying in geosynchronous orbit where the tension is the greatest. @Mark’s suggestion that martian elevators are achievable while Earth elevators are out of reach is also an excellent explanation for the proposed situation.
                  – Mike Nichols
                  Aug 9 at 21:39






                • 2




                  No terrorists needed -- 1000 years from now, materials science has advanced to the point building a space elevator on Mars is trivial, but on Earth is still practically impossible: physics.stackexchange.com/questions/33547/…
                  – Alexander Duchene
                  Aug 10 at 3:05










                • I understand some ideas about space elevators include an "anchor" at the top, like an asteroid in geosynchronous orbit. So perhaps that could have crashed.
                  – Joe
                  Aug 10 at 22:01







                3




                3




                While space elevators are horribly vulnerable it's only an economic catastrophe if one comes down. The cables aren't going to be heavy enough to do anything serious if they do come down (and that's assuming they don't simply burn as they fall, anyway.)
                – Loren Pechtel
                Aug 9 at 21:18




                While space elevators are horribly vulnerable it's only an economic catastrophe if one comes down. The cables aren't going to be heavy enough to do anything serious if they do come down (and that's assuming they don't simply burn as they fall, anyway.)
                – Loren Pechtel
                Aug 9 at 21:18




                8




                8




                Any realistic elevator cable is going to be more ribbon-like than rope-like, so a severed cable isn't going to come crashing down. The reason Mars has a space elevator and Earth doesn't is much simpler: lower gravity. A space elevator on Earth requires technobabble materials to build, while a Martian elevator could probably be done using Kevlar or carbon fiber.
                – Mark
                Aug 9 at 21:28




                Any realistic elevator cable is going to be more ribbon-like than rope-like, so a severed cable isn't going to come crashing down. The reason Mars has a space elevator and Earth doesn't is much simpler: lower gravity. A space elevator on Earth requires technobabble materials to build, while a Martian elevator could probably be done using Kevlar or carbon fiber.
                – Mark
                Aug 9 at 21:28




                3




                3




                I would caution that since we don’t know what material these elevators will be made of we shouldn’t discount the possibility that they will be heavy enough to do damage. Perhaps unlikely, but not implausible. Also worth noting that the cable will get thicker as you go up with the thickest portion lying in geosynchronous orbit where the tension is the greatest. @Mark’s suggestion that martian elevators are achievable while Earth elevators are out of reach is also an excellent explanation for the proposed situation.
                – Mike Nichols
                Aug 9 at 21:39




                I would caution that since we don’t know what material these elevators will be made of we shouldn’t discount the possibility that they will be heavy enough to do damage. Perhaps unlikely, but not implausible. Also worth noting that the cable will get thicker as you go up with the thickest portion lying in geosynchronous orbit where the tension is the greatest. @Mark’s suggestion that martian elevators are achievable while Earth elevators are out of reach is also an excellent explanation for the proposed situation.
                – Mike Nichols
                Aug 9 at 21:39




                2




                2




                No terrorists needed -- 1000 years from now, materials science has advanced to the point building a space elevator on Mars is trivial, but on Earth is still practically impossible: physics.stackexchange.com/questions/33547/…
                – Alexander Duchene
                Aug 10 at 3:05




                No terrorists needed -- 1000 years from now, materials science has advanced to the point building a space elevator on Mars is trivial, but on Earth is still practically impossible: physics.stackexchange.com/questions/33547/…
                – Alexander Duchene
                Aug 10 at 3:05












                I understand some ideas about space elevators include an "anchor" at the top, like an asteroid in geosynchronous orbit. So perhaps that could have crashed.
                – Joe
                Aug 10 at 22:01




                I understand some ideas about space elevators include an "anchor" at the top, like an asteroid in geosynchronous orbit. So perhaps that could have crashed.
                – Joe
                Aug 10 at 22:01










                up vote
                7
                down vote













                Honestly, I don't see that you have a problem



                How many people own yachts? And yet the cheapest form of transport on Earth is an ocean-going super freighter. So, from the perspective of people just jumping in their cars and moving, there's already the problem of the cost of the car spaceship, cost of fuel, and travel time. Your average Joe Schmoe could never afford the trip.



                How to keep the wealthy off? That's easy, too. Mars property (somewhat like Hawaii) is 100% leased to corporate ventures under strict laws and oversight by a panel of United Earth nations who manage the resource. There isn't a square inch available for non-purpose habitation and the consequences are, well, let's call them breathtaking in their scope of civil punishment.



                Note that you'll never have 100% compliance. You'll have the corporations creating resorts for their people, friends, and "associates." All well within the UE rules, of course. But that kind of nearly meaninless population isn't, I think, what you're talking about.



                Finally, just in case you decide to make space travel cheap. If it is cheap, then so is the policing force needed to beat away the unwashed barbarians who don't want to live within spitting distance of their Terran neighbors.






                share|improve this answer
























                  up vote
                  7
                  down vote













                  Honestly, I don't see that you have a problem



                  How many people own yachts? And yet the cheapest form of transport on Earth is an ocean-going super freighter. So, from the perspective of people just jumping in their cars and moving, there's already the problem of the cost of the car spaceship, cost of fuel, and travel time. Your average Joe Schmoe could never afford the trip.



                  How to keep the wealthy off? That's easy, too. Mars property (somewhat like Hawaii) is 100% leased to corporate ventures under strict laws and oversight by a panel of United Earth nations who manage the resource. There isn't a square inch available for non-purpose habitation and the consequences are, well, let's call them breathtaking in their scope of civil punishment.



                  Note that you'll never have 100% compliance. You'll have the corporations creating resorts for their people, friends, and "associates." All well within the UE rules, of course. But that kind of nearly meaninless population isn't, I think, what you're talking about.



                  Finally, just in case you decide to make space travel cheap. If it is cheap, then so is the policing force needed to beat away the unwashed barbarians who don't want to live within spitting distance of their Terran neighbors.






                  share|improve this answer






















                    up vote
                    7
                    down vote










                    up vote
                    7
                    down vote









                    Honestly, I don't see that you have a problem



                    How many people own yachts? And yet the cheapest form of transport on Earth is an ocean-going super freighter. So, from the perspective of people just jumping in their cars and moving, there's already the problem of the cost of the car spaceship, cost of fuel, and travel time. Your average Joe Schmoe could never afford the trip.



                    How to keep the wealthy off? That's easy, too. Mars property (somewhat like Hawaii) is 100% leased to corporate ventures under strict laws and oversight by a panel of United Earth nations who manage the resource. There isn't a square inch available for non-purpose habitation and the consequences are, well, let's call them breathtaking in their scope of civil punishment.



                    Note that you'll never have 100% compliance. You'll have the corporations creating resorts for their people, friends, and "associates." All well within the UE rules, of course. But that kind of nearly meaninless population isn't, I think, what you're talking about.



                    Finally, just in case you decide to make space travel cheap. If it is cheap, then so is the policing force needed to beat away the unwashed barbarians who don't want to live within spitting distance of their Terran neighbors.






                    share|improve this answer












                    Honestly, I don't see that you have a problem



                    How many people own yachts? And yet the cheapest form of transport on Earth is an ocean-going super freighter. So, from the perspective of people just jumping in their cars and moving, there's already the problem of the cost of the car spaceship, cost of fuel, and travel time. Your average Joe Schmoe could never afford the trip.



                    How to keep the wealthy off? That's easy, too. Mars property (somewhat like Hawaii) is 100% leased to corporate ventures under strict laws and oversight by a panel of United Earth nations who manage the resource. There isn't a square inch available for non-purpose habitation and the consequences are, well, let's call them breathtaking in their scope of civil punishment.



                    Note that you'll never have 100% compliance. You'll have the corporations creating resorts for their people, friends, and "associates." All well within the UE rules, of course. But that kind of nearly meaninless population isn't, I think, what you're talking about.



                    Finally, just in case you decide to make space travel cheap. If it is cheap, then so is the policing force needed to beat away the unwashed barbarians who don't want to live within spitting distance of their Terran neighbors.







                    share|improve this answer












                    share|improve this answer



                    share|improve this answer










                    answered Aug 9 at 20:07









                    JBH

                    31.1k573147




                    31.1k573147




















                        up vote
                        7
                        down vote













                        No technobabble required. Earth has a deep gravity well and a huge, thick atmosphere. Mars has a shallow gravity well, and presumably even after terraforming still has a thin atmosphere.



                        Going from Mars to Earth is cheap (if a bit slow). A surface-mounted mass driver can throw packages of logs directly into an Earth-intercept trajectory. The thin atmosphere means you don't need much shielding on the way out (particularly if you're launching from the top of Olympus Mons), and Earth's thick atmosphere provides easy aerocapture and landing six months later.



                        Going from Earth to Mars is much harder. The accelerations involved in a mass-driver launch make it unsuitable for anything but durable freight; humans and fragile goods need to be launched on chemical rockets. Either way, you're spending far more energy fighting gravity and drag than you would leaving Mars. At the other end of the six-month trip (which itself would tend to discourage human travel), the thin Martian atmosphere makes aerocapture and parachute/glider landing tricky. You'll need to haul engines and fuel with you, which further drives up the cost of travel.






                        share|improve this answer
























                          up vote
                          7
                          down vote













                          No technobabble required. Earth has a deep gravity well and a huge, thick atmosphere. Mars has a shallow gravity well, and presumably even after terraforming still has a thin atmosphere.



                          Going from Mars to Earth is cheap (if a bit slow). A surface-mounted mass driver can throw packages of logs directly into an Earth-intercept trajectory. The thin atmosphere means you don't need much shielding on the way out (particularly if you're launching from the top of Olympus Mons), and Earth's thick atmosphere provides easy aerocapture and landing six months later.



                          Going from Earth to Mars is much harder. The accelerations involved in a mass-driver launch make it unsuitable for anything but durable freight; humans and fragile goods need to be launched on chemical rockets. Either way, you're spending far more energy fighting gravity and drag than you would leaving Mars. At the other end of the six-month trip (which itself would tend to discourage human travel), the thin Martian atmosphere makes aerocapture and parachute/glider landing tricky. You'll need to haul engines and fuel with you, which further drives up the cost of travel.






                          share|improve this answer






















                            up vote
                            7
                            down vote










                            up vote
                            7
                            down vote









                            No technobabble required. Earth has a deep gravity well and a huge, thick atmosphere. Mars has a shallow gravity well, and presumably even after terraforming still has a thin atmosphere.



                            Going from Mars to Earth is cheap (if a bit slow). A surface-mounted mass driver can throw packages of logs directly into an Earth-intercept trajectory. The thin atmosphere means you don't need much shielding on the way out (particularly if you're launching from the top of Olympus Mons), and Earth's thick atmosphere provides easy aerocapture and landing six months later.



                            Going from Earth to Mars is much harder. The accelerations involved in a mass-driver launch make it unsuitable for anything but durable freight; humans and fragile goods need to be launched on chemical rockets. Either way, you're spending far more energy fighting gravity and drag than you would leaving Mars. At the other end of the six-month trip (which itself would tend to discourage human travel), the thin Martian atmosphere makes aerocapture and parachute/glider landing tricky. You'll need to haul engines and fuel with you, which further drives up the cost of travel.






                            share|improve this answer












                            No technobabble required. Earth has a deep gravity well and a huge, thick atmosphere. Mars has a shallow gravity well, and presumably even after terraforming still has a thin atmosphere.



                            Going from Mars to Earth is cheap (if a bit slow). A surface-mounted mass driver can throw packages of logs directly into an Earth-intercept trajectory. The thin atmosphere means you don't need much shielding on the way out (particularly if you're launching from the top of Olympus Mons), and Earth's thick atmosphere provides easy aerocapture and landing six months later.



                            Going from Earth to Mars is much harder. The accelerations involved in a mass-driver launch make it unsuitable for anything but durable freight; humans and fragile goods need to be launched on chemical rockets. Either way, you're spending far more energy fighting gravity and drag than you would leaving Mars. At the other end of the six-month trip (which itself would tend to discourage human travel), the thin Martian atmosphere makes aerocapture and parachute/glider landing tricky. You'll need to haul engines and fuel with you, which further drives up the cost of travel.







                            share|improve this answer












                            share|improve this answer



                            share|improve this answer










                            answered Aug 9 at 22:25









                            Mark

                            11.6k2856




                            11.6k2856




















                                up vote
                                6
                                down vote













                                Your antigravity drives or whatever you are using work only when the body has a bulk density comparable to wood, about $0.8 g/cm^3$. Anything denser than that will have to follow the chemical rocket path, with its related costs, efficiencies and limitations.



                                Casually enough this will allow you to cheaply ship wood, but nothing more than that.






                                share|improve this answer


















                                • 2




                                  Potassium conveniently has a density comparable to wood(0.86), so if Mars does have ancient evaporated seas then an earlier potassium/potash/fertilizer boom could be the reason why Mars is forested in the first place.
                                  – Giter
                                  Aug 9 at 20:04







                                • 4




                                  Fridge logic: Air is much less dense than wood, you should have no trouble shipping a load of steel if you only partially fill the spacecraft...
                                  – Brilliand
                                  Aug 9 at 22:04






                                • 1




                                  @Brilliand: Perfect, now I have an excuse to design heavier-than-air zeppelins.
                                  – Kevin
                                  Aug 10 at 0:58










                                • @Brilliand, writing my answer I had in mind the bulk density, not the average density. I have now edited my answer to make it clear.
                                  – L.Dutch♦
                                  Aug 10 at 6:19














                                up vote
                                6
                                down vote













                                Your antigravity drives or whatever you are using work only when the body has a bulk density comparable to wood, about $0.8 g/cm^3$. Anything denser than that will have to follow the chemical rocket path, with its related costs, efficiencies and limitations.



                                Casually enough this will allow you to cheaply ship wood, but nothing more than that.






                                share|improve this answer


















                                • 2




                                  Potassium conveniently has a density comparable to wood(0.86), so if Mars does have ancient evaporated seas then an earlier potassium/potash/fertilizer boom could be the reason why Mars is forested in the first place.
                                  – Giter
                                  Aug 9 at 20:04







                                • 4




                                  Fridge logic: Air is much less dense than wood, you should have no trouble shipping a load of steel if you only partially fill the spacecraft...
                                  – Brilliand
                                  Aug 9 at 22:04






                                • 1




                                  @Brilliand: Perfect, now I have an excuse to design heavier-than-air zeppelins.
                                  – Kevin
                                  Aug 10 at 0:58










                                • @Brilliand, writing my answer I had in mind the bulk density, not the average density. I have now edited my answer to make it clear.
                                  – L.Dutch♦
                                  Aug 10 at 6:19












                                up vote
                                6
                                down vote










                                up vote
                                6
                                down vote









                                Your antigravity drives or whatever you are using work only when the body has a bulk density comparable to wood, about $0.8 g/cm^3$. Anything denser than that will have to follow the chemical rocket path, with its related costs, efficiencies and limitations.



                                Casually enough this will allow you to cheaply ship wood, but nothing more than that.






                                share|improve this answer














                                Your antigravity drives or whatever you are using work only when the body has a bulk density comparable to wood, about $0.8 g/cm^3$. Anything denser than that will have to follow the chemical rocket path, with its related costs, efficiencies and limitations.



                                Casually enough this will allow you to cheaply ship wood, but nothing more than that.







                                share|improve this answer














                                share|improve this answer



                                share|improve this answer








                                edited Aug 10 at 3:16

























                                answered Aug 9 at 19:43









                                L.Dutch♦

                                59.1k15138278




                                59.1k15138278







                                • 2




                                  Potassium conveniently has a density comparable to wood(0.86), so if Mars does have ancient evaporated seas then an earlier potassium/potash/fertilizer boom could be the reason why Mars is forested in the first place.
                                  – Giter
                                  Aug 9 at 20:04







                                • 4




                                  Fridge logic: Air is much less dense than wood, you should have no trouble shipping a load of steel if you only partially fill the spacecraft...
                                  – Brilliand
                                  Aug 9 at 22:04






                                • 1




                                  @Brilliand: Perfect, now I have an excuse to design heavier-than-air zeppelins.
                                  – Kevin
                                  Aug 10 at 0:58










                                • @Brilliand, writing my answer I had in mind the bulk density, not the average density. I have now edited my answer to make it clear.
                                  – L.Dutch♦
                                  Aug 10 at 6:19












                                • 2




                                  Potassium conveniently has a density comparable to wood(0.86), so if Mars does have ancient evaporated seas then an earlier potassium/potash/fertilizer boom could be the reason why Mars is forested in the first place.
                                  – Giter
                                  Aug 9 at 20:04







                                • 4




                                  Fridge logic: Air is much less dense than wood, you should have no trouble shipping a load of steel if you only partially fill the spacecraft...
                                  – Brilliand
                                  Aug 9 at 22:04






                                • 1




                                  @Brilliand: Perfect, now I have an excuse to design heavier-than-air zeppelins.
                                  – Kevin
                                  Aug 10 at 0:58










                                • @Brilliand, writing my answer I had in mind the bulk density, not the average density. I have now edited my answer to make it clear.
                                  – L.Dutch♦
                                  Aug 10 at 6:19







                                2




                                2




                                Potassium conveniently has a density comparable to wood(0.86), so if Mars does have ancient evaporated seas then an earlier potassium/potash/fertilizer boom could be the reason why Mars is forested in the first place.
                                – Giter
                                Aug 9 at 20:04





                                Potassium conveniently has a density comparable to wood(0.86), so if Mars does have ancient evaporated seas then an earlier potassium/potash/fertilizer boom could be the reason why Mars is forested in the first place.
                                – Giter
                                Aug 9 at 20:04





                                4




                                4




                                Fridge logic: Air is much less dense than wood, you should have no trouble shipping a load of steel if you only partially fill the spacecraft...
                                – Brilliand
                                Aug 9 at 22:04




                                Fridge logic: Air is much less dense than wood, you should have no trouble shipping a load of steel if you only partially fill the spacecraft...
                                – Brilliand
                                Aug 9 at 22:04




                                1




                                1




                                @Brilliand: Perfect, now I have an excuse to design heavier-than-air zeppelins.
                                – Kevin
                                Aug 10 at 0:58




                                @Brilliand: Perfect, now I have an excuse to design heavier-than-air zeppelins.
                                – Kevin
                                Aug 10 at 0:58












                                @Brilliand, writing my answer I had in mind the bulk density, not the average density. I have now edited my answer to make it clear.
                                – L.Dutch♦
                                Aug 10 at 6:19




                                @Brilliand, writing my answer I had in mind the bulk density, not the average density. I have now edited my answer to make it clear.
                                – L.Dutch♦
                                Aug 10 at 6:19










                                up vote
                                4
                                down vote













                                Material limits. While we can't currently build a space elevator on Mars we can do the next best thing--elevators on Deimos and Phobos. (See here) Now, we don't need much more strength to get the low end of the Phobos elevator into the range that aircraft can intercept it.



                                Presto, travel from Mars to Earth becomes cheap, the hardest part of the whole mission is protecting them from their trip through the fire coming down. However, the cables aren't strong enough to build an Earth-based elevator. Thus to go from Earth to Mars still has to be done the hard way on a tail of fire.



                                You're not going to get it down to loggers with hand tools as the launch from Mars still needs high performance aircraft--but that's still an awful lot cheaper than rockets.






                                share|improve this answer


























                                  up vote
                                  4
                                  down vote













                                  Material limits. While we can't currently build a space elevator on Mars we can do the next best thing--elevators on Deimos and Phobos. (See here) Now, we don't need much more strength to get the low end of the Phobos elevator into the range that aircraft can intercept it.



                                  Presto, travel from Mars to Earth becomes cheap, the hardest part of the whole mission is protecting them from their trip through the fire coming down. However, the cables aren't strong enough to build an Earth-based elevator. Thus to go from Earth to Mars still has to be done the hard way on a tail of fire.



                                  You're not going to get it down to loggers with hand tools as the launch from Mars still needs high performance aircraft--but that's still an awful lot cheaper than rockets.






                                  share|improve this answer
























                                    up vote
                                    4
                                    down vote










                                    up vote
                                    4
                                    down vote









                                    Material limits. While we can't currently build a space elevator on Mars we can do the next best thing--elevators on Deimos and Phobos. (See here) Now, we don't need much more strength to get the low end of the Phobos elevator into the range that aircraft can intercept it.



                                    Presto, travel from Mars to Earth becomes cheap, the hardest part of the whole mission is protecting them from their trip through the fire coming down. However, the cables aren't strong enough to build an Earth-based elevator. Thus to go from Earth to Mars still has to be done the hard way on a tail of fire.



                                    You're not going to get it down to loggers with hand tools as the launch from Mars still needs high performance aircraft--but that's still an awful lot cheaper than rockets.






                                    share|improve this answer














                                    Material limits. While we can't currently build a space elevator on Mars we can do the next best thing--elevators on Deimos and Phobos. (See here) Now, we don't need much more strength to get the low end of the Phobos elevator into the range that aircraft can intercept it.



                                    Presto, travel from Mars to Earth becomes cheap, the hardest part of the whole mission is protecting them from their trip through the fire coming down. However, the cables aren't strong enough to build an Earth-based elevator. Thus to go from Earth to Mars still has to be done the hard way on a tail of fire.



                                    You're not going to get it down to loggers with hand tools as the launch from Mars still needs high performance aircraft--but that's still an awful lot cheaper than rockets.







                                    share|improve this answer














                                    share|improve this answer



                                    share|improve this answer








                                    edited Aug 11 at 2:58

























                                    answered Aug 9 at 21:17









                                    Loren Pechtel

                                    17.4k2153




                                    17.4k2153




















                                        up vote
                                        3
                                        down vote













                                        Terraforming without lots of tech.



                                        Later this year, astronomers discover a very large icy comet inbound from the Kuiper belt, destined to make impact with an Asteroid so that it scrubs most of the velocity relative to Mars. The remnants from this collision impact Mars 2 months later delivering many megatons of water, and other interesting volatiles such as carbon dioxide, carbon monoxide, and ammonia. Won't be pleasant on Mars for quite a while, but once things settle, you have the basic materials you need in place.



                                        Taking advantage of this event, a seed ship is sent to Mars after the fact to seed the new atmosphere with various designed to transform the atmosphere into something suitable for forestry.



                                        Still, Mars has trouble retaining an atmosphere, so NASA also prepares a big magnetic shield for the atmosphere to help prevent atmosphere from being stripped away by the solar wind.



                                        Making it comparatively cheap to send wood back to Earth requires non-fanciful technologies. A space elevator would be much easier to construct for Mars than Earth because of its lower gravity. Still won't be cheap to build, but you don't have to start construction for a long-time still -- waiting for better materials, construction techniques, etc.



                                        Significant improvements in orbital transfer (between Mars and Earth) are also needed. Careful use of light sails should make this reasonably priced in either direction as long as you are not in a hurry.



                                        Dropping wood from orbit to the Earth is not that difficult if you are willing to use the wood itself as ablative shielding.






                                        share|improve this answer




















                                        • I think the suggestion of building space elevators on mars is a good one - that's something much easier there than earth.
                                          – Tim B♦
                                          Aug 9 at 20:54














                                        up vote
                                        3
                                        down vote













                                        Terraforming without lots of tech.



                                        Later this year, astronomers discover a very large icy comet inbound from the Kuiper belt, destined to make impact with an Asteroid so that it scrubs most of the velocity relative to Mars. The remnants from this collision impact Mars 2 months later delivering many megatons of water, and other interesting volatiles such as carbon dioxide, carbon monoxide, and ammonia. Won't be pleasant on Mars for quite a while, but once things settle, you have the basic materials you need in place.



                                        Taking advantage of this event, a seed ship is sent to Mars after the fact to seed the new atmosphere with various designed to transform the atmosphere into something suitable for forestry.



                                        Still, Mars has trouble retaining an atmosphere, so NASA also prepares a big magnetic shield for the atmosphere to help prevent atmosphere from being stripped away by the solar wind.



                                        Making it comparatively cheap to send wood back to Earth requires non-fanciful technologies. A space elevator would be much easier to construct for Mars than Earth because of its lower gravity. Still won't be cheap to build, but you don't have to start construction for a long-time still -- waiting for better materials, construction techniques, etc.



                                        Significant improvements in orbital transfer (between Mars and Earth) are also needed. Careful use of light sails should make this reasonably priced in either direction as long as you are not in a hurry.



                                        Dropping wood from orbit to the Earth is not that difficult if you are willing to use the wood itself as ablative shielding.






                                        share|improve this answer




















                                        • I think the suggestion of building space elevators on mars is a good one - that's something much easier there than earth.
                                          – Tim B♦
                                          Aug 9 at 20:54












                                        up vote
                                        3
                                        down vote










                                        up vote
                                        3
                                        down vote









                                        Terraforming without lots of tech.



                                        Later this year, astronomers discover a very large icy comet inbound from the Kuiper belt, destined to make impact with an Asteroid so that it scrubs most of the velocity relative to Mars. The remnants from this collision impact Mars 2 months later delivering many megatons of water, and other interesting volatiles such as carbon dioxide, carbon monoxide, and ammonia. Won't be pleasant on Mars for quite a while, but once things settle, you have the basic materials you need in place.



                                        Taking advantage of this event, a seed ship is sent to Mars after the fact to seed the new atmosphere with various designed to transform the atmosphere into something suitable for forestry.



                                        Still, Mars has trouble retaining an atmosphere, so NASA also prepares a big magnetic shield for the atmosphere to help prevent atmosphere from being stripped away by the solar wind.



                                        Making it comparatively cheap to send wood back to Earth requires non-fanciful technologies. A space elevator would be much easier to construct for Mars than Earth because of its lower gravity. Still won't be cheap to build, but you don't have to start construction for a long-time still -- waiting for better materials, construction techniques, etc.



                                        Significant improvements in orbital transfer (between Mars and Earth) are also needed. Careful use of light sails should make this reasonably priced in either direction as long as you are not in a hurry.



                                        Dropping wood from orbit to the Earth is not that difficult if you are willing to use the wood itself as ablative shielding.






                                        share|improve this answer












                                        Terraforming without lots of tech.



                                        Later this year, astronomers discover a very large icy comet inbound from the Kuiper belt, destined to make impact with an Asteroid so that it scrubs most of the velocity relative to Mars. The remnants from this collision impact Mars 2 months later delivering many megatons of water, and other interesting volatiles such as carbon dioxide, carbon monoxide, and ammonia. Won't be pleasant on Mars for quite a while, but once things settle, you have the basic materials you need in place.



                                        Taking advantage of this event, a seed ship is sent to Mars after the fact to seed the new atmosphere with various designed to transform the atmosphere into something suitable for forestry.



                                        Still, Mars has trouble retaining an atmosphere, so NASA also prepares a big magnetic shield for the atmosphere to help prevent atmosphere from being stripped away by the solar wind.



                                        Making it comparatively cheap to send wood back to Earth requires non-fanciful technologies. A space elevator would be much easier to construct for Mars than Earth because of its lower gravity. Still won't be cheap to build, but you don't have to start construction for a long-time still -- waiting for better materials, construction techniques, etc.



                                        Significant improvements in orbital transfer (between Mars and Earth) are also needed. Careful use of light sails should make this reasonably priced in either direction as long as you are not in a hurry.



                                        Dropping wood from orbit to the Earth is not that difficult if you are willing to use the wood itself as ablative shielding.







                                        share|improve this answer












                                        share|improve this answer



                                        share|improve this answer










                                        answered Aug 9 at 20:17









                                        Gary Walker

                                        14.2k22753




                                        14.2k22753











                                        • I think the suggestion of building space elevators on mars is a good one - that's something much easier there than earth.
                                          – Tim B♦
                                          Aug 9 at 20:54
















                                        • I think the suggestion of building space elevators on mars is a good one - that's something much easier there than earth.
                                          – Tim B♦
                                          Aug 9 at 20:54















                                        I think the suggestion of building space elevators on mars is a good one - that's something much easier there than earth.
                                        – Tim B♦
                                        Aug 9 at 20:54




                                        I think the suggestion of building space elevators on mars is a good one - that's something much easier there than earth.
                                        – Tim B♦
                                        Aug 9 at 20:54










                                        up vote
                                        3
                                        down vote













                                        The wood itself is the fuel for the rockets.



                                        In combination with jedmeyer's answer suggesting that the trees grow much larger than they would on Earth, perhaps there is a real chemical process or, failing that, something catalyzed by handwavium (mined only on Mars) that allows the wood itself to function as rocket fuel. Hollow out a large tree, fill it with oxidizer, and launch it. It could fire retrorockets on its approach to Earth and splash down in the ocean where it would be recovered by a fleet of logging ships.



                                        (I know this answer is absurd, but it amuses me greatly.)






                                        share|improve this answer




















                                        • Welcome to WorldBuilding.SE! We're glad you could join us! I don't know how to put links in my comments, or else I would invite you to click somewhere to learn about our rules and culture, or take a tour. I'm sure someone will fill that in for me. This is a fun answer :) The uses for handwavium are as abundant as the resource itself. Hope you enjoy your time here!
                                          – boxcartenant
                                          Aug 9 at 20:51










                                        • It seems like this would decrease both the quality and the quantity of the wood received, not to mention the wood would probably just burn up during reentry. But it is quite the creative idea.
                                          – Billy S
                                          Aug 9 at 23:06










                                        • @BillyS, wood is a decent material to make an ablative heat shield out of. It's considerably heavier than synthetic resin heatshields, which is why it never saw much use, but if you're already throwing logs from planet to planet...
                                          – Mark
                                          Aug 10 at 0:47










                                        • @Mark Really? I wouldn't have expected that, due to wood's flammable nature ... I guess I should do more research next time.
                                          – Billy S
                                          Aug 10 at 1:13







                                        • 1




                                          @BillyS, wood tends to form an outer layer of char that provides decent insulation and resists burning. It takes quite a bit of effort to actually burn a full-sized log.
                                          – Mark
                                          Aug 10 at 1:52














                                        up vote
                                        3
                                        down vote













                                        The wood itself is the fuel for the rockets.



                                        In combination with jedmeyer's answer suggesting that the trees grow much larger than they would on Earth, perhaps there is a real chemical process or, failing that, something catalyzed by handwavium (mined only on Mars) that allows the wood itself to function as rocket fuel. Hollow out a large tree, fill it with oxidizer, and launch it. It could fire retrorockets on its approach to Earth and splash down in the ocean where it would be recovered by a fleet of logging ships.



                                        (I know this answer is absurd, but it amuses me greatly.)






                                        share|improve this answer




















                                        • Welcome to WorldBuilding.SE! We're glad you could join us! I don't know how to put links in my comments, or else I would invite you to click somewhere to learn about our rules and culture, or take a tour. I'm sure someone will fill that in for me. This is a fun answer :) The uses for handwavium are as abundant as the resource itself. Hope you enjoy your time here!
                                          – boxcartenant
                                          Aug 9 at 20:51










                                        • It seems like this would decrease both the quality and the quantity of the wood received, not to mention the wood would probably just burn up during reentry. But it is quite the creative idea.
                                          – Billy S
                                          Aug 9 at 23:06










                                        • @BillyS, wood is a decent material to make an ablative heat shield out of. It's considerably heavier than synthetic resin heatshields, which is why it never saw much use, but if you're already throwing logs from planet to planet...
                                          – Mark
                                          Aug 10 at 0:47










                                        • @Mark Really? I wouldn't have expected that, due to wood's flammable nature ... I guess I should do more research next time.
                                          – Billy S
                                          Aug 10 at 1:13







                                        • 1




                                          @BillyS, wood tends to form an outer layer of char that provides decent insulation and resists burning. It takes quite a bit of effort to actually burn a full-sized log.
                                          – Mark
                                          Aug 10 at 1:52












                                        up vote
                                        3
                                        down vote










                                        up vote
                                        3
                                        down vote









                                        The wood itself is the fuel for the rockets.



                                        In combination with jedmeyer's answer suggesting that the trees grow much larger than they would on Earth, perhaps there is a real chemical process or, failing that, something catalyzed by handwavium (mined only on Mars) that allows the wood itself to function as rocket fuel. Hollow out a large tree, fill it with oxidizer, and launch it. It could fire retrorockets on its approach to Earth and splash down in the ocean where it would be recovered by a fleet of logging ships.



                                        (I know this answer is absurd, but it amuses me greatly.)






                                        share|improve this answer












                                        The wood itself is the fuel for the rockets.



                                        In combination with jedmeyer's answer suggesting that the trees grow much larger than they would on Earth, perhaps there is a real chemical process or, failing that, something catalyzed by handwavium (mined only on Mars) that allows the wood itself to function as rocket fuel. Hollow out a large tree, fill it with oxidizer, and launch it. It could fire retrorockets on its approach to Earth and splash down in the ocean where it would be recovered by a fleet of logging ships.



                                        (I know this answer is absurd, but it amuses me greatly.)







                                        share|improve this answer












                                        share|improve this answer



                                        share|improve this answer










                                        answered Aug 9 at 20:39









                                        TKK

                                        1393




                                        1393











                                        • Welcome to WorldBuilding.SE! We're glad you could join us! I don't know how to put links in my comments, or else I would invite you to click somewhere to learn about our rules and culture, or take a tour. I'm sure someone will fill that in for me. This is a fun answer :) The uses for handwavium are as abundant as the resource itself. Hope you enjoy your time here!
                                          – boxcartenant
                                          Aug 9 at 20:51










                                        • It seems like this would decrease both the quality and the quantity of the wood received, not to mention the wood would probably just burn up during reentry. But it is quite the creative idea.
                                          – Billy S
                                          Aug 9 at 23:06










                                        • @BillyS, wood is a decent material to make an ablative heat shield out of. It's considerably heavier than synthetic resin heatshields, which is why it never saw much use, but if you're already throwing logs from planet to planet...
                                          – Mark
                                          Aug 10 at 0:47










                                        • @Mark Really? I wouldn't have expected that, due to wood's flammable nature ... I guess I should do more research next time.
                                          – Billy S
                                          Aug 10 at 1:13







                                        • 1




                                          @BillyS, wood tends to form an outer layer of char that provides decent insulation and resists burning. It takes quite a bit of effort to actually burn a full-sized log.
                                          – Mark
                                          Aug 10 at 1:52
















                                        • Welcome to WorldBuilding.SE! We're glad you could join us! I don't know how to put links in my comments, or else I would invite you to click somewhere to learn about our rules and culture, or take a tour. I'm sure someone will fill that in for me. This is a fun answer :) The uses for handwavium are as abundant as the resource itself. Hope you enjoy your time here!
                                          – boxcartenant
                                          Aug 9 at 20:51










                                        • It seems like this would decrease both the quality and the quantity of the wood received, not to mention the wood would probably just burn up during reentry. But it is quite the creative idea.
                                          – Billy S
                                          Aug 9 at 23:06










                                        • @BillyS, wood is a decent material to make an ablative heat shield out of. It's considerably heavier than synthetic resin heatshields, which is why it never saw much use, but if you're already throwing logs from planet to planet...
                                          – Mark
                                          Aug 10 at 0:47










                                        • @Mark Really? I wouldn't have expected that, due to wood's flammable nature ... I guess I should do more research next time.
                                          – Billy S
                                          Aug 10 at 1:13







                                        • 1




                                          @BillyS, wood tends to form an outer layer of char that provides decent insulation and resists burning. It takes quite a bit of effort to actually burn a full-sized log.
                                          – Mark
                                          Aug 10 at 1:52















                                        Welcome to WorldBuilding.SE! We're glad you could join us! I don't know how to put links in my comments, or else I would invite you to click somewhere to learn about our rules and culture, or take a tour. I'm sure someone will fill that in for me. This is a fun answer :) The uses for handwavium are as abundant as the resource itself. Hope you enjoy your time here!
                                        – boxcartenant
                                        Aug 9 at 20:51




                                        Welcome to WorldBuilding.SE! We're glad you could join us! I don't know how to put links in my comments, or else I would invite you to click somewhere to learn about our rules and culture, or take a tour. I'm sure someone will fill that in for me. This is a fun answer :) The uses for handwavium are as abundant as the resource itself. Hope you enjoy your time here!
                                        – boxcartenant
                                        Aug 9 at 20:51












                                        It seems like this would decrease both the quality and the quantity of the wood received, not to mention the wood would probably just burn up during reentry. But it is quite the creative idea.
                                        – Billy S
                                        Aug 9 at 23:06




                                        It seems like this would decrease both the quality and the quantity of the wood received, not to mention the wood would probably just burn up during reentry. But it is quite the creative idea.
                                        – Billy S
                                        Aug 9 at 23:06












                                        @BillyS, wood is a decent material to make an ablative heat shield out of. It's considerably heavier than synthetic resin heatshields, which is why it never saw much use, but if you're already throwing logs from planet to planet...
                                        – Mark
                                        Aug 10 at 0:47




                                        @BillyS, wood is a decent material to make an ablative heat shield out of. It's considerably heavier than synthetic resin heatshields, which is why it never saw much use, but if you're already throwing logs from planet to planet...
                                        – Mark
                                        Aug 10 at 0:47












                                        @Mark Really? I wouldn't have expected that, due to wood's flammable nature ... I guess I should do more research next time.
                                        – Billy S
                                        Aug 10 at 1:13





                                        @Mark Really? I wouldn't have expected that, due to wood's flammable nature ... I guess I should do more research next time.
                                        – Billy S
                                        Aug 10 at 1:13





                                        1




                                        1




                                        @BillyS, wood tends to form an outer layer of char that provides decent insulation and resists burning. It takes quite a bit of effort to actually burn a full-sized log.
                                        – Mark
                                        Aug 10 at 1:52




                                        @BillyS, wood tends to form an outer layer of char that provides decent insulation and resists burning. It takes quite a bit of effort to actually burn a full-sized log.
                                        – Mark
                                        Aug 10 at 1:52










                                        up vote
                                        2
                                        down vote













                                        The power of Economics!



                                        The solution you have here is that Mars has less gravity than Earth (3.7m/s^2 vs. (9.8 m/s^2). Because of this massive difference, the trees that grew up on Earth are able to grow MUCH taller on Mars, solving the scenario in two ways.



                                        1. The trees grow faster and taller due to less gravity. Thus, a frontier-like world could develop shortly after being seeded and "terraformed".

                                        2. Big Trees = More Wood. If you grow trees easier on Mars, Martian wood becomes more affordable: supply and demand. The shipping is just an overhead cost.

                                        Where the scenario gets more challenging is the "downstream" metaphor. It would be possible if you consider aerobraking, since Mars has a less dense atmosphere than Earth. Launching from Mars requires a little bit less to land on Earth thanks to its thick atmosphere than the reverse path. However, that makes it unlikely that megaflora could flourish due to the insufficient pressure.



                                        I recommend that you consider another cause for the unindustrialized world:



                                        -There isn't an established mining base on Mars for enough metal for automated machinery, and no one wants to ship that out when you get enough wood from the locals!



                                        -Overpopulation on Earth means more affordable labor for companies, and there's no room to plant large forests!






                                        share|improve this answer




















                                        • Part of what I'm thinking about to explain why few people go to Mars is that people on Earth will generally be wimps. The risk of splinters, the loud noises, the weather that isn't programmed on a schedule, will all seem too rugged to most of them.
                                          – Joe
                                          Aug 10 at 22:13














                                        up vote
                                        2
                                        down vote













                                        The power of Economics!



                                        The solution you have here is that Mars has less gravity than Earth (3.7m/s^2 vs. (9.8 m/s^2). Because of this massive difference, the trees that grew up on Earth are able to grow MUCH taller on Mars, solving the scenario in two ways.



                                        1. The trees grow faster and taller due to less gravity. Thus, a frontier-like world could develop shortly after being seeded and "terraformed".

                                        2. Big Trees = More Wood. If you grow trees easier on Mars, Martian wood becomes more affordable: supply and demand. The shipping is just an overhead cost.

                                        Where the scenario gets more challenging is the "downstream" metaphor. It would be possible if you consider aerobraking, since Mars has a less dense atmosphere than Earth. Launching from Mars requires a little bit less to land on Earth thanks to its thick atmosphere than the reverse path. However, that makes it unlikely that megaflora could flourish due to the insufficient pressure.



                                        I recommend that you consider another cause for the unindustrialized world:



                                        -There isn't an established mining base on Mars for enough metal for automated machinery, and no one wants to ship that out when you get enough wood from the locals!



                                        -Overpopulation on Earth means more affordable labor for companies, and there's no room to plant large forests!






                                        share|improve this answer




















                                        • Part of what I'm thinking about to explain why few people go to Mars is that people on Earth will generally be wimps. The risk of splinters, the loud noises, the weather that isn't programmed on a schedule, will all seem too rugged to most of them.
                                          – Joe
                                          Aug 10 at 22:13












                                        up vote
                                        2
                                        down vote










                                        up vote
                                        2
                                        down vote









                                        The power of Economics!



                                        The solution you have here is that Mars has less gravity than Earth (3.7m/s^2 vs. (9.8 m/s^2). Because of this massive difference, the trees that grew up on Earth are able to grow MUCH taller on Mars, solving the scenario in two ways.



                                        1. The trees grow faster and taller due to less gravity. Thus, a frontier-like world could develop shortly after being seeded and "terraformed".

                                        2. Big Trees = More Wood. If you grow trees easier on Mars, Martian wood becomes more affordable: supply and demand. The shipping is just an overhead cost.

                                        Where the scenario gets more challenging is the "downstream" metaphor. It would be possible if you consider aerobraking, since Mars has a less dense atmosphere than Earth. Launching from Mars requires a little bit less to land on Earth thanks to its thick atmosphere than the reverse path. However, that makes it unlikely that megaflora could flourish due to the insufficient pressure.



                                        I recommend that you consider another cause for the unindustrialized world:



                                        -There isn't an established mining base on Mars for enough metal for automated machinery, and no one wants to ship that out when you get enough wood from the locals!



                                        -Overpopulation on Earth means more affordable labor for companies, and there's no room to plant large forests!






                                        share|improve this answer












                                        The power of Economics!



                                        The solution you have here is that Mars has less gravity than Earth (3.7m/s^2 vs. (9.8 m/s^2). Because of this massive difference, the trees that grew up on Earth are able to grow MUCH taller on Mars, solving the scenario in two ways.



                                        1. The trees grow faster and taller due to less gravity. Thus, a frontier-like world could develop shortly after being seeded and "terraformed".

                                        2. Big Trees = More Wood. If you grow trees easier on Mars, Martian wood becomes more affordable: supply and demand. The shipping is just an overhead cost.

                                        Where the scenario gets more challenging is the "downstream" metaphor. It would be possible if you consider aerobraking, since Mars has a less dense atmosphere than Earth. Launching from Mars requires a little bit less to land on Earth thanks to its thick atmosphere than the reverse path. However, that makes it unlikely that megaflora could flourish due to the insufficient pressure.



                                        I recommend that you consider another cause for the unindustrialized world:



                                        -There isn't an established mining base on Mars for enough metal for automated machinery, and no one wants to ship that out when you get enough wood from the locals!



                                        -Overpopulation on Earth means more affordable labor for companies, and there's no room to plant large forests!







                                        share|improve this answer












                                        share|improve this answer



                                        share|improve this answer










                                        answered Aug 9 at 20:06









                                        jedmeyer

                                        45317




                                        45317











                                        • Part of what I'm thinking about to explain why few people go to Mars is that people on Earth will generally be wimps. The risk of splinters, the loud noises, the weather that isn't programmed on a schedule, will all seem too rugged to most of them.
                                          – Joe
                                          Aug 10 at 22:13
















                                        • Part of what I'm thinking about to explain why few people go to Mars is that people on Earth will generally be wimps. The risk of splinters, the loud noises, the weather that isn't programmed on a schedule, will all seem too rugged to most of them.
                                          – Joe
                                          Aug 10 at 22:13















                                        Part of what I'm thinking about to explain why few people go to Mars is that people on Earth will generally be wimps. The risk of splinters, the loud noises, the weather that isn't programmed on a schedule, will all seem too rugged to most of them.
                                        – Joe
                                        Aug 10 at 22:13




                                        Part of what I'm thinking about to explain why few people go to Mars is that people on Earth will generally be wimps. The risk of splinters, the loud noises, the weather that isn't programmed on a schedule, will all seem too rugged to most of them.
                                        – Joe
                                        Aug 10 at 22:13










                                        up vote
                                        2
                                        down vote













                                        I like @Mark's mass driver for logs - bare logs flying thru space.



                                        My idea for the cheap low tech terraforming - sometime in the mid 21st century the popular prophet inheriting the American government (and its nuclear stock) is told by God that nuking the Martian polar icecaps will bring great rewards to humanity. The Americans proceed to do exactly that, launching hundreds of ICBMs to detonate on Mars. The nukes are followed with arklike rockets of seeds and spores.



                                        The scheme works like a charm, liberating immense stores of water and CO2 which cause a runaway greenhouse effect and melt the rest of the immense subsurface stores of these gases. Certain seeds among those sent up give rise to the forests which are thriving 1000 years later.






                                        share|improve this answer
























                                          up vote
                                          2
                                          down vote













                                          I like @Mark's mass driver for logs - bare logs flying thru space.



                                          My idea for the cheap low tech terraforming - sometime in the mid 21st century the popular prophet inheriting the American government (and its nuclear stock) is told by God that nuking the Martian polar icecaps will bring great rewards to humanity. The Americans proceed to do exactly that, launching hundreds of ICBMs to detonate on Mars. The nukes are followed with arklike rockets of seeds and spores.



                                          The scheme works like a charm, liberating immense stores of water and CO2 which cause a runaway greenhouse effect and melt the rest of the immense subsurface stores of these gases. Certain seeds among those sent up give rise to the forests which are thriving 1000 years later.






                                          share|improve this answer






















                                            up vote
                                            2
                                            down vote










                                            up vote
                                            2
                                            down vote









                                            I like @Mark's mass driver for logs - bare logs flying thru space.



                                            My idea for the cheap low tech terraforming - sometime in the mid 21st century the popular prophet inheriting the American government (and its nuclear stock) is told by God that nuking the Martian polar icecaps will bring great rewards to humanity. The Americans proceed to do exactly that, launching hundreds of ICBMs to detonate on Mars. The nukes are followed with arklike rockets of seeds and spores.



                                            The scheme works like a charm, liberating immense stores of water and CO2 which cause a runaway greenhouse effect and melt the rest of the immense subsurface stores of these gases. Certain seeds among those sent up give rise to the forests which are thriving 1000 years later.






                                            share|improve this answer












                                            I like @Mark's mass driver for logs - bare logs flying thru space.



                                            My idea for the cheap low tech terraforming - sometime in the mid 21st century the popular prophet inheriting the American government (and its nuclear stock) is told by God that nuking the Martian polar icecaps will bring great rewards to humanity. The Americans proceed to do exactly that, launching hundreds of ICBMs to detonate on Mars. The nukes are followed with arklike rockets of seeds and spores.



                                            The scheme works like a charm, liberating immense stores of water and CO2 which cause a runaway greenhouse effect and melt the rest of the immense subsurface stores of these gases. Certain seeds among those sent up give rise to the forests which are thriving 1000 years later.







                                            share|improve this answer












                                            share|improve this answer



                                            share|improve this answer










                                            answered Aug 9 at 22:50









                                            Willk

                                            83.8k20165360




                                            83.8k20165360




















                                                up vote
                                                2
                                                down vote













                                                Mars has lower gravity than Earth, so it's already easier to achieve escape velocity.



                                                However, with all of Mars as a giant forest, the atmosphere is now much more oxygen-rich than Earth. This means that producing liquid oxygen fuel is quicker and cheaper, and any craft with an air-intake can burn fuel much more efficiently than one on Earth.



                                                In that circumstance, Earth really would be downstream. The freighter's easily lift into Mars orbit, burn the rest of their fuel to escape orbit at just the right spot...and free-fall into a precise Earth orbit where "local" spacecraft can retrieve them and bring them down.



                                                You could even add a touch of danger to the retrieval: if the freight isn't caught and slowed on the first pass, it'll be going too fast, and will slingshot into empty space. If a corp misses a shipment at the prime capture point, nearby opportunists could turn a hefty profit by claiming the freight later in its trajectory--when it's much more dangerous to make the attempt.






                                                share|improve this answer
























                                                  up vote
                                                  2
                                                  down vote













                                                  Mars has lower gravity than Earth, so it's already easier to achieve escape velocity.



                                                  However, with all of Mars as a giant forest, the atmosphere is now much more oxygen-rich than Earth. This means that producing liquid oxygen fuel is quicker and cheaper, and any craft with an air-intake can burn fuel much more efficiently than one on Earth.



                                                  In that circumstance, Earth really would be downstream. The freighter's easily lift into Mars orbit, burn the rest of their fuel to escape orbit at just the right spot...and free-fall into a precise Earth orbit where "local" spacecraft can retrieve them and bring them down.



                                                  You could even add a touch of danger to the retrieval: if the freight isn't caught and slowed on the first pass, it'll be going too fast, and will slingshot into empty space. If a corp misses a shipment at the prime capture point, nearby opportunists could turn a hefty profit by claiming the freight later in its trajectory--when it's much more dangerous to make the attempt.






                                                  share|improve this answer






















                                                    up vote
                                                    2
                                                    down vote










                                                    up vote
                                                    2
                                                    down vote









                                                    Mars has lower gravity than Earth, so it's already easier to achieve escape velocity.



                                                    However, with all of Mars as a giant forest, the atmosphere is now much more oxygen-rich than Earth. This means that producing liquid oxygen fuel is quicker and cheaper, and any craft with an air-intake can burn fuel much more efficiently than one on Earth.



                                                    In that circumstance, Earth really would be downstream. The freighter's easily lift into Mars orbit, burn the rest of their fuel to escape orbit at just the right spot...and free-fall into a precise Earth orbit where "local" spacecraft can retrieve them and bring them down.



                                                    You could even add a touch of danger to the retrieval: if the freight isn't caught and slowed on the first pass, it'll be going too fast, and will slingshot into empty space. If a corp misses a shipment at the prime capture point, nearby opportunists could turn a hefty profit by claiming the freight later in its trajectory--when it's much more dangerous to make the attempt.






                                                    share|improve this answer












                                                    Mars has lower gravity than Earth, so it's already easier to achieve escape velocity.



                                                    However, with all of Mars as a giant forest, the atmosphere is now much more oxygen-rich than Earth. This means that producing liquid oxygen fuel is quicker and cheaper, and any craft with an air-intake can burn fuel much more efficiently than one on Earth.



                                                    In that circumstance, Earth really would be downstream. The freighter's easily lift into Mars orbit, burn the rest of their fuel to escape orbit at just the right spot...and free-fall into a precise Earth orbit where "local" spacecraft can retrieve them and bring them down.



                                                    You could even add a touch of danger to the retrieval: if the freight isn't caught and slowed on the first pass, it'll be going too fast, and will slingshot into empty space. If a corp misses a shipment at the prime capture point, nearby opportunists could turn a hefty profit by claiming the freight later in its trajectory--when it's much more dangerous to make the attempt.







                                                    share|improve this answer












                                                    share|improve this answer



                                                    share|improve this answer










                                                    answered Aug 10 at 0:25









                                                    Liesmith

                                                    3,82121024




                                                    3,82121024




















                                                        up vote
                                                        2
                                                        down vote













                                                        Monsanto got chased off of Earth



                                                        Monsanto is one of the most hated big corporations on this planet presently. Not because their products are actually bad; genetically modified crops are safe and pretty much ubiquitous by now. But Monsanto's business practices has every ideologically green party and ditto non governmental organisation getting their knickers in a big itchy twist. And somewhat rightfully so because Monsanto is really pushing the limits when it comes to copyrights, trying to control what people can do with their products long after the product has been bought and used by the client.



                                                        So Monsanto — or its 3018 equivalent — have set up shop on Mars instead. And they are doing quite well for themselves! Their radiation hardened planktons and tree products have been instrumental in terraforming Mars without having to bother with restoring the planet's magnetic field. And not only that but the wood is amazing, being (insert qualities that are great for wood to have here). There is no-one that has those species of trees growing anywhere, thanks to Monsanto's quite excellent gene-hackers/engineers.



                                                        Of course there is the slight issue on how to protect such a valuable product and not have it spread like weed all over the solar system and neighbouring stars. So the genetic code is written such that it requires Mars conditions — the right gravity, the right radiation levels, the right atmosphere — or the seeds / seedlings simply will not catch. And Monsanto are annoyingly good at this sort of genetic copyright protection.



                                                        Too bad for them then, when an ideologically green terrorist organisation infiltrated the Ice Miner's Guild and in a brilliant move bot-net'd an entire flotilla of harvest drones to annihilate every Monsanto office on Mars by dropping asteroid ice bergs on them. (Side plot hook: what happens when you take a genetic engineering corporation, physically smash it to bits and let out all their lab samples and secret projects on a young booming forest planet)



                                                        So there you have it: a — now defunct — genetics engineering corporation created a marvellous but very Mars specific species of wood... and then they got wiped out. Now their products keep growing on Mars, and are ripe for harvest.






                                                        share|improve this answer


























                                                          up vote
                                                          2
                                                          down vote













                                                          Monsanto got chased off of Earth



                                                          Monsanto is one of the most hated big corporations on this planet presently. Not because their products are actually bad; genetically modified crops are safe and pretty much ubiquitous by now. But Monsanto's business practices has every ideologically green party and ditto non governmental organisation getting their knickers in a big itchy twist. And somewhat rightfully so because Monsanto is really pushing the limits when it comes to copyrights, trying to control what people can do with their products long after the product has been bought and used by the client.



                                                          So Monsanto — or its 3018 equivalent — have set up shop on Mars instead. And they are doing quite well for themselves! Their radiation hardened planktons and tree products have been instrumental in terraforming Mars without having to bother with restoring the planet's magnetic field. And not only that but the wood is amazing, being (insert qualities that are great for wood to have here). There is no-one that has those species of trees growing anywhere, thanks to Monsanto's quite excellent gene-hackers/engineers.



                                                          Of course there is the slight issue on how to protect such a valuable product and not have it spread like weed all over the solar system and neighbouring stars. So the genetic code is written such that it requires Mars conditions — the right gravity, the right radiation levels, the right atmosphere — or the seeds / seedlings simply will not catch. And Monsanto are annoyingly good at this sort of genetic copyright protection.



                                                          Too bad for them then, when an ideologically green terrorist organisation infiltrated the Ice Miner's Guild and in a brilliant move bot-net'd an entire flotilla of harvest drones to annihilate every Monsanto office on Mars by dropping asteroid ice bergs on them. (Side plot hook: what happens when you take a genetic engineering corporation, physically smash it to bits and let out all their lab samples and secret projects on a young booming forest planet)



                                                          So there you have it: a — now defunct — genetics engineering corporation created a marvellous but very Mars specific species of wood... and then they got wiped out. Now their products keep growing on Mars, and are ripe for harvest.






                                                          share|improve this answer
























                                                            up vote
                                                            2
                                                            down vote










                                                            up vote
                                                            2
                                                            down vote









                                                            Monsanto got chased off of Earth



                                                            Monsanto is one of the most hated big corporations on this planet presently. Not because their products are actually bad; genetically modified crops are safe and pretty much ubiquitous by now. But Monsanto's business practices has every ideologically green party and ditto non governmental organisation getting their knickers in a big itchy twist. And somewhat rightfully so because Monsanto is really pushing the limits when it comes to copyrights, trying to control what people can do with their products long after the product has been bought and used by the client.



                                                            So Monsanto — or its 3018 equivalent — have set up shop on Mars instead. And they are doing quite well for themselves! Their radiation hardened planktons and tree products have been instrumental in terraforming Mars without having to bother with restoring the planet's magnetic field. And not only that but the wood is amazing, being (insert qualities that are great for wood to have here). There is no-one that has those species of trees growing anywhere, thanks to Monsanto's quite excellent gene-hackers/engineers.



                                                            Of course there is the slight issue on how to protect such a valuable product and not have it spread like weed all over the solar system and neighbouring stars. So the genetic code is written such that it requires Mars conditions — the right gravity, the right radiation levels, the right atmosphere — or the seeds / seedlings simply will not catch. And Monsanto are annoyingly good at this sort of genetic copyright protection.



                                                            Too bad for them then, when an ideologically green terrorist organisation infiltrated the Ice Miner's Guild and in a brilliant move bot-net'd an entire flotilla of harvest drones to annihilate every Monsanto office on Mars by dropping asteroid ice bergs on them. (Side plot hook: what happens when you take a genetic engineering corporation, physically smash it to bits and let out all their lab samples and secret projects on a young booming forest planet)



                                                            So there you have it: a — now defunct — genetics engineering corporation created a marvellous but very Mars specific species of wood... and then they got wiped out. Now their products keep growing on Mars, and are ripe for harvest.






                                                            share|improve this answer














                                                            Monsanto got chased off of Earth



                                                            Monsanto is one of the most hated big corporations on this planet presently. Not because their products are actually bad; genetically modified crops are safe and pretty much ubiquitous by now. But Monsanto's business practices has every ideologically green party and ditto non governmental organisation getting their knickers in a big itchy twist. And somewhat rightfully so because Monsanto is really pushing the limits when it comes to copyrights, trying to control what people can do with their products long after the product has been bought and used by the client.



                                                            So Monsanto — or its 3018 equivalent — have set up shop on Mars instead. And they are doing quite well for themselves! Their radiation hardened planktons and tree products have been instrumental in terraforming Mars without having to bother with restoring the planet's magnetic field. And not only that but the wood is amazing, being (insert qualities that are great for wood to have here). There is no-one that has those species of trees growing anywhere, thanks to Monsanto's quite excellent gene-hackers/engineers.



                                                            Of course there is the slight issue on how to protect such a valuable product and not have it spread like weed all over the solar system and neighbouring stars. So the genetic code is written such that it requires Mars conditions — the right gravity, the right radiation levels, the right atmosphere — or the seeds / seedlings simply will not catch. And Monsanto are annoyingly good at this sort of genetic copyright protection.



                                                            Too bad for them then, when an ideologically green terrorist organisation infiltrated the Ice Miner's Guild and in a brilliant move bot-net'd an entire flotilla of harvest drones to annihilate every Monsanto office on Mars by dropping asteroid ice bergs on them. (Side plot hook: what happens when you take a genetic engineering corporation, physically smash it to bits and let out all their lab samples and secret projects on a young booming forest planet)



                                                            So there you have it: a — now defunct — genetics engineering corporation created a marvellous but very Mars specific species of wood... and then they got wiped out. Now their products keep growing on Mars, and are ripe for harvest.







                                                            share|improve this answer














                                                            share|improve this answer



                                                            share|improve this answer








                                                            edited Aug 10 at 7:38

























                                                            answered Aug 10 at 6:21









                                                            MichaelK

                                                            33.8k580143




                                                            33.8k580143




















                                                                up vote
                                                                1
                                                                down vote













                                                                Terraforming Mars: solar-powered artificial magnetosphere, water from Ceres (Mars is downstream from Ceres and ceres lower gravity makes it easy to fly water to Mars), nitrogen from Titan's atmosphere to start the Nitrogen cycle. In the long term, some fertilizers in mineral form (and additional Carbon Dioxide) must be replenished in one way or another.



                                                                Environmental issues: too much damage to ecosystems on Earth is a good motive to turn Mars into a huge nature reserve with plants, wildlife and other organisms. Migration to Mars is restricted to park rangers, loggers and other related jobs, so that population is minimal. Some area is designated for logging.



                                                                Self-government: the new Martians take control of their business. Machinery is restricted to less sophisticated tools, but cargo spaceships are still indispensable. Park rangers may double as loggers in the spare time and further reduce the need for more workers. The "martians" may actually make lots of profits from managing this business, and controlling who & how others join is a way to keep much of the profit to themselves. They may windowdress their management decisions with environmentalist ideology.






                                                                share|improve this answer


























                                                                  up vote
                                                                  1
                                                                  down vote













                                                                  Terraforming Mars: solar-powered artificial magnetosphere, water from Ceres (Mars is downstream from Ceres and ceres lower gravity makes it easy to fly water to Mars), nitrogen from Titan's atmosphere to start the Nitrogen cycle. In the long term, some fertilizers in mineral form (and additional Carbon Dioxide) must be replenished in one way or another.



                                                                  Environmental issues: too much damage to ecosystems on Earth is a good motive to turn Mars into a huge nature reserve with plants, wildlife and other organisms. Migration to Mars is restricted to park rangers, loggers and other related jobs, so that population is minimal. Some area is designated for logging.



                                                                  Self-government: the new Martians take control of their business. Machinery is restricted to less sophisticated tools, but cargo spaceships are still indispensable. Park rangers may double as loggers in the spare time and further reduce the need for more workers. The "martians" may actually make lots of profits from managing this business, and controlling who & how others join is a way to keep much of the profit to themselves. They may windowdress their management decisions with environmentalist ideology.






                                                                  share|improve this answer
























                                                                    up vote
                                                                    1
                                                                    down vote










                                                                    up vote
                                                                    1
                                                                    down vote









                                                                    Terraforming Mars: solar-powered artificial magnetosphere, water from Ceres (Mars is downstream from Ceres and ceres lower gravity makes it easy to fly water to Mars), nitrogen from Titan's atmosphere to start the Nitrogen cycle. In the long term, some fertilizers in mineral form (and additional Carbon Dioxide) must be replenished in one way or another.



                                                                    Environmental issues: too much damage to ecosystems on Earth is a good motive to turn Mars into a huge nature reserve with plants, wildlife and other organisms. Migration to Mars is restricted to park rangers, loggers and other related jobs, so that population is minimal. Some area is designated for logging.



                                                                    Self-government: the new Martians take control of their business. Machinery is restricted to less sophisticated tools, but cargo spaceships are still indispensable. Park rangers may double as loggers in the spare time and further reduce the need for more workers. The "martians" may actually make lots of profits from managing this business, and controlling who & how others join is a way to keep much of the profit to themselves. They may windowdress their management decisions with environmentalist ideology.






                                                                    share|improve this answer














                                                                    Terraforming Mars: solar-powered artificial magnetosphere, water from Ceres (Mars is downstream from Ceres and ceres lower gravity makes it easy to fly water to Mars), nitrogen from Titan's atmosphere to start the Nitrogen cycle. In the long term, some fertilizers in mineral form (and additional Carbon Dioxide) must be replenished in one way or another.



                                                                    Environmental issues: too much damage to ecosystems on Earth is a good motive to turn Mars into a huge nature reserve with plants, wildlife and other organisms. Migration to Mars is restricted to park rangers, loggers and other related jobs, so that population is minimal. Some area is designated for logging.



                                                                    Self-government: the new Martians take control of their business. Machinery is restricted to less sophisticated tools, but cargo spaceships are still indispensable. Park rangers may double as loggers in the spare time and further reduce the need for more workers. The "martians" may actually make lots of profits from managing this business, and controlling who & how others join is a way to keep much of the profit to themselves. They may windowdress their management decisions with environmentalist ideology.







                                                                    share|improve this answer














                                                                    share|improve this answer



                                                                    share|improve this answer








                                                                    edited Aug 10 at 5:34

























                                                                    answered Aug 9 at 20:13









                                                                    Christmas Snow

                                                                    1,25329




                                                                    1,25329






















                                                                         

                                                                        draft saved


                                                                        draft discarded


























                                                                         


                                                                        draft saved


                                                                        draft discarded














                                                                        StackExchange.ready(
                                                                        function ()
                                                                        StackExchange.openid.initPostLogin('.new-post-login', 'https%3a%2f%2fworldbuilding.stackexchange.com%2fquestions%2f120907%2fwhats-a-plausible-sounding-technobabble-explanation-for-mars-to-be-a-forest-wor%23new-answer', 'question_page');

                                                                        );

                                                                        Post as a guest













































































                                                                        這個網誌中的熱門文章

                                                                        Is there any way to eliminate the singular point to solve this integral by hand or by approximations?

                                                                        Why am i infinitely getting the same tweet with the Twitter Search API?

                                                                        Carbon dioxide